[obm-l] Re: [obm-l] Re: [obm-l] Combinatória

2024-03-13 Por tôpico Pedro Júnior
Eu pensei sim, mas e os casos do tipo ACCACAC. Esse caso não entra na conta
6! - 2* 3!* 3!.

Em qua., 13 de mar. de 2024 às 09:09, Claudio Buffara <
claudio.buff...@gmail.com> escreveu:

> Pense no oposto: de quantas maneiras as crianças e adultos podem se sentar
> separados uns dos outros.
>
> On Wed, Mar 13, 2024 at 8:39 AM Pedro Júnior 
> wrote:
>
>> Olá pessoal, bom dia.
>> Alguém poderia me ajudar nesse problema?
>>
>> Seis poltronas enfileiradas em um cinema e entram 3 adultos e 3 crianças.
>> De quantas maneiras podem sentar-se 2 crianças juntas e dois adultos juntos?
>>
>>
>> Desde já fico grato!
>>
>> --
>> Esta mensagem foi verificada pelo sistema de antivírus e
>> acredita-se estar livre de perigo.
>
>
> --
> Esta mensagem foi verificada pelo sistema de antivírus e
> acredita-se estar livre de perigo.



-- 

Pedro Jerônimo S. de O. Júnior

Professor de Matemática

Geo João Pessoa – PB

-- 
Esta mensagem foi verificada pelo sistema de antiv�rus e
 acredita-se estar livre de perigo.



[obm-l] Combinatória

2024-03-13 Por tôpico Pedro Júnior
Olá pessoal, bom dia.
Alguém poderia me ajudar nesse problema?

Seis poltronas enfileiradas em um cinema e entram 3 adultos e 3 crianças.
De quantas maneiras podem sentar-se 2 crianças juntas e dois adultos juntos?


Desde já fico grato!

-- 
Esta mensagem foi verificada pelo sistema de antiv�rus e
 acredita-se estar livre de perigo.



[obm-l] Re: [obm-l] Re: [obm-l] Desigualdade e frações

2024-02-27 Por tôpico Pedro Júnior
Desculpas, Cláudio. É isso mesmo, com "a" e "b" inteiros e positivos.

Obrigado pela brilhante solução.

Em ter, 27 de fev de 2024 01:41, Claudio Buffara 
escreveu:

> Deveria ser a e b inteiros positivos, não?
> Pois se forem inteiros sem restrição, então como 2022/2023 < 2022,5/2023,5
> < 2023/2024, bastaria tomar a sequência:
> a(n) = -20225*n  e  b(n) = -20235*n.
> Daí teríamos 2022/2023 < a(n)/b(n) < 2023/2024 e a sequência a(n)+b(n)
> seria ilimitada inferiormente.
>
> Assim, suponhamos que a e b sejam inteiros positivos.
> 2022/2023 < a/b < 2023/2024 implica que b > a+1, já que a sequência
> (n/(n+1)) é crescente.
> Além disso, usando razões e proporções, achamos que:
> 2022 < a/(b-a) < 2023 < b/(b-a) < 2024
> ==> para que a+b seja o menor possível, b-a deverá ser o menor possível.
> E o menor valor possível de b-a é 2.
> Usando frações equivalentes, dá pra escrever 4044/4046 < a/b < 4046/4048 e
> daí teríamos uma única fração a/b com b - a = 2.
> Seria a/b = 4045/4047 ==> a+b mínimo = 8092.
>
> []s,
> Claudio.
>
>
>
>
> On Mon, Feb 26, 2024 at 10:12 PM Pedro Júnior 
> wrote:
>
>> Quem puder me ajudar, fixo grato.
>>
>> Sejam a e b dois números inteiros. Sabendo que 2022/2023 < a/b <
>> 2023/2024, determine o menos calor da soma a + b.
>>
>> --
>> Esta mensagem foi verificada pelo sistema de antivírus e
>> acredita-se estar livre de perigo.
>
>
> --
> Esta mensagem foi verificada pelo sistema de antivírus e
> acredita-se estar livre de perigo.

-- 
Esta mensagem foi verificada pelo sistema de antiv�rus e
 acredita-se estar livre de perigo.



[obm-l] Desigualdade e frações

2024-02-26 Por tôpico Pedro Júnior
Quem puder me ajudar, fixo grato.

Sejam a e b dois números inteiros. Sabendo que 2022/2023 < a/b < 2023/2024,
determine o menos calor da soma a + b.

-- 
Esta mensagem foi verificada pelo sistema de antiv�rus e
 acredita-se estar livre de perigo.



[obm-l] Cone Sul

2023-03-13 Por tôpico Pedro Júnior
Olá pessoal, muito bom dia.
Gostaria de saber se tem um site oficial da competição "Cone Sul de
Matemática"? Procurei o banco de provas pelo Google e não encontrei. Me
remete ao site da OBM e também não vi por lá.

Desde já fico grato.

-- 
Esta mensagem foi verificada pelo sistema de antiv�rus e
 acredita-se estar livre de perigo.



[obm-l] Re: [obm-l] Re: [obm-l] Invertíveis e Divisores de Zero

2021-11-30 Por tôpico Pedro Júnior
Sim...

Em ter., 30 de nov. de 2021 às 15:21, Claudio Buffara <
claudio.buff...@gmail.com> escreveu:

> Z_4 x Z_5 é isomorfo a Z_20.
> Talvez isso ajude.
>
> On Tue, Nov 30, 2021 at 2:33 PM Pedro Júnior 
> wrote:
>
>> Quem puder ajudar...
>> Encontre todos os invertíveis e divisores de zero em Z_4 x Z_5.
>>
>>
>>
>> --
>> Esta mensagem foi verificada pelo sistema de antivírus e
>> acredita-se estar livre de perigo.
>
>
> --
> Esta mensagem foi verificada pelo sistema de antivírus e
> acredita-se estar livre de perigo.



-- 

Pedro Jerônimo S. de O. Júnior

Professor de Matemática

Geo João Pessoa – PB

-- 
Esta mensagem foi verificada pelo sistema de antiv�rus e
 acredita-se estar livre de perigo.



[obm-l] Invertíveis e Divisores de Zero

2021-11-30 Por tôpico Pedro Júnior
Quem puder ajudar...
Encontre todos os invertíveis e divisores de zero em Z_4 x Z_5.

-- 
Esta mensagem foi verificada pelo sistema de antiv�rus e
 acredita-se estar livre de perigo.



Re: [obm-l] OBMEP 2021 - Fase 2 - N3

2021-11-09 Por tôpico Pedro Júnior
Obrigado, Ralph!

Em ter., 9 de nov. de 2021 às 13:21, Ralph Costa Teixeira 
escreveu:

> Suponho que (A) e (B) sejam fáceis -- basta seguir o algoritmo na mão e
> ver o que acontece.
>
> Para facilitar a conversa, vou pensar em "tempo" como o número de
> movimentos feitos... Ou seja, o tempo 0 corresponde à posição inicial; o
> tempo 1 seria logo após o primeiro movimento; etc.
>
> Para (C), pense assim: se o sistema tem alguma coroa no tempo (n), eu
> afirmo que vai ter alguma coroa no tempo (n+1). De fato:
> -- Se o ponteiro aponta para uma cara no tempo (n), o sistema não muda, e
> a tal coroa continua ali;
> -- Se o ponteiro aponta para uma coroa no tempo (n), ESTA coroa vai ficar
> presente no tempo (n+1).
> Portanto, sempre teremos coroas.
> (Talvez seja mais natural pensar assim: como que o sistema passaria de
> "ter coroas" para "não ter coroas"? Bom, para ele mudar o ponteiro tem que
> apontar para alguma coroa, e esta coroa NÃO MUDA. Ou seja,
> impossível passar de "ter coroas" para "não ter coroas".)
>
> Para (D), note que o sistema tem apenas (2^10) * 10 configurações
> possíveis (o número não interessa tanto, o que importa é que é FINITO; note
> que incluo ali as posições das moedas E a do ponteiro), enquanto o tempo
> avança sempre, então em algum momento alguma configuração vai ter que
> repetir.
> Mas pense como "desfazer" o último movimento realizado e você vai perceber
> que existe apenas um jeito de "voltar no tempo" (deixo para você descrever
> exatamente isso)! Ou seja, o sistema é reversível (olhando como ficou o
> sistema no tempo (n+1), você consegue deduzir como ele estava no tempo (n),
> revertendo o último movimento, de maneira única). Portanto, se o sistema
> tinha a mesma configuração nos tempos A e A+T, revertendo os movimentos,
> concluímos que vai ter a mesma configuração nos tempos 0 e T; ou seja, no
> tempo T tínhamos todas coroas como no tempo 0 (e o ponteiro apontando para
> A! Bônus!)
>
> Abraço, Ralph.
>
> On Tue, Nov 9, 2021 at 12:22 PM Pedro Júnior 
> wrote:
>
>> Olá pessoal, alguém aí conseguiu fazer essa questão da prova da OBMEP
>> 2021 N3, fase 2? Se puder, ajuda aí... Valeu!
>>
>> 6) há 10 moedas em um círculo nomeadas de A a J, inicialmente todas com a
>> face coroa virada para cima. No centro desse círculo, há um ponteiro que
>> inicialmente aponta para a moeda A. Esse ponteiro se movimenta, girando no
>> sentido anti-horário (A->B->C->...->J->A->...). Ao movimentar-se, há duas
>> opções:
>> •Quando o ponteiro termina o movimento apontando para uma moeda com a
>> face coroa virada para cima, a moeda seguinte é virada.
>> •Quando o ponteiro termina o movimento apontando para uma moeda com a
>> face cara virada para cima, nada acontece.
>>
>> Há exemplo, no primeiro movimento (de A para B), o ponteiro termina em B,
>> e assim, vira-se a moeda C, que fica com a face cara para cima.
>>
>> Letra A) o que acontece com as moedas C e D após o segundo movimento?
>>
>> Letra B) Depois do 12º movimento, quais moedas estão com a face coroa
>> virada para cima?
>>
>> Letra C) mostre que é impossível que, após certo número de movimentos,
>> todas as moedas fiquem com a face cara para cima.
>>
>> Letra D) Mostre que, após um certo número de movimentos, todas as moedas
>> voltarão a ficar com a face coroa para cima.
>>
>>
>>
>> --
>> Esta mensagem foi verificada pelo sistema de antivírus e
>> acredita-se estar livre de perigo.
>
>
> --
> Esta mensagem foi verificada pelo sistema de antivírus e
> acredita-se estar livre de perigo.



-- 

Pedro Jerônimo S. de O. Júnior

Professor de Matemática

Geo João Pessoa – PB

-- 
Esta mensagem foi verificada pelo sistema de antiv�rus e
 acredita-se estar livre de perigo.



[obm-l] OBMEP 2021 - Fase 2 - N3

2021-11-09 Por tôpico Pedro Júnior
Olá pessoal, alguém aí conseguiu fazer essa questão da prova da OBMEP 2021
N3, fase 2? Se puder, ajuda aí... Valeu!

6) há 10 moedas em um círculo nomeadas de A a J, inicialmente todas com a
face coroa virada para cima. No centro desse círculo, há um ponteiro que
inicialmente aponta para a moeda A. Esse ponteiro se movimenta, girando no
sentido anti-horário (A->B->C->...->J->A->...). Ao movimentar-se, há duas
opções:
•Quando o ponteiro termina o movimento apontando para uma moeda com a face
coroa virada para cima, a moeda seguinte é virada.
•Quando o ponteiro termina o movimento apontando para uma moeda com a face
cara virada para cima, nada acontece.

Há exemplo, no primeiro movimento (de A para B), o ponteiro termina em B, e
assim, vira-se a moeda C, que fica com a face cara para cima.

Letra A) o que acontece com as moedas C e D após o segundo movimento?

Letra B) Depois do 12º movimento, quais moedas estão com a face coroa
virada para cima?

Letra C) mostre que é impossível que, após certo número de movimentos,
todas as moedas fiquem com a face cara para cima.

Letra D) Mostre que, após um certo número de movimentos, todas as moedas
voltarão a ficar com a face coroa para cima.

-- 
Esta mensagem foi verificada pelo sistema de antiv�rus e
 acredita-se estar livre de perigo.



[obm-l] Re: [obm-l] RES: [obm-l] Re: [obm-l] INFLAÇÂO MÁXIMA

2021-04-24 Por tôpico Pedro Júnior
Sobre a gramática, verdade!
Sobre a Matemática, impecável.
Consegui ver onde eu estava errando.

Obrigado, professor.

Em sex, 23 de abr de 2021 14:24,  escreveu:

> Olá!
>
> Para começar, esta questão deveria ter sido anulada. “… não HAJAM perdas
> reais?” é um assassinato da nossa língua.
>
>
>
> Juros “reais” (JR), de 10%, significam juros acima da inflação (IF).
>
> No período de 1 ano, o ganho bruto de capital (GB) será: GB = 1.000
> (1+10%)(1+IF) - 1.000
>
> Descontando o imposto, o ganho líquido (GL) será: GL =  (1-40%)GB
>
> Condição de contorno: não “hão” (coerência com o linguajar da questão)
> perdas reais: 1.000(1+IF) = 1.000+GL = 1.000 + (1-40%)GB = 1.000 + (1-40%)(
> 1.000 (1+10%)(1+IF) - 1.000 )
>
> Daí: IF = 17,6470…%
>
>
>
> *Albert Bouskelá*
>
> bousk...@gmail.com
>
>
>
> *De:* owner-ob...@mat.puc-rio.br  *Em nome de
> *Daniel Jelin
> *Enviada em:* sexta-feira, 23 de abril de 2021 12:30
> *Para:* obm-l@mat.puc-rio.br
> *Assunto:* [obm-l] Re: [obm-l] INFLAÇÂO MÁXIMA
>
>
>
> Curioso, pra mim deu muito perto, 17,6470...%
>
> Resolvi a seguinte inequação, com x = 1 + (inflação):
>
> 1.1*1000x - (1.1*1000x - 1000)*0.4>=1000x
> 1.1 x - 0.44 x + 0.4 >= x
> x<=0.4/0.34= 1.176470...
>
> Parece simples. O que tá escapando aqui?
>
>
>
> On Fri, Apr 23, 2021 at 11:23 AM Pedro Júnior 
> wrote:
>
> Olá pessoal, acabei me enrolando nesse probleminha da Olimpíada Brasileira
> de Economia. Será que alguém pode me ajudar? Vai junto o gabarito da
> competição, isso foi em 2020.
>
>
>
> *01)* Um título comprado por mil reais promete pagar juros reais de 10%
> a.a. A alíquota de imposto é de 40%. Qual a inflação máxima no período para
> que não hajam perdas reais?
>
> Resp.: 17,62%
>
>
>
> --
>
> Pedro Jerônimo S. de O. Júnior
>
> Professor de Matemática
>
>
>
>
> --
> Esta mensagem foi verificada pelo sistema de antivírus e
> acredita-se estar livre de perigo.
>
>
> --
> Esta mensagem foi verificada pelo sistema de antiv?s e
> acredita-se estar livre de perigo.
>
> --
> Esta mensagem foi verificada pelo sistema de antivírus e
> acredita-se estar livre de perigo.
>

-- 
Esta mensagem foi verificada pelo sistema de antiv�rus e
 acredita-se estar livre de perigo.



[obm-l] INFLAÇÂO MÁXIMA

2021-04-23 Por tôpico Pedro Júnior
Olá pessoal, acabei me enrolando nesse probleminha da Olimpíada Brasileira
de Economia. Será que alguém pode me ajudar? Vai junto o gabarito da
competição, isso foi em 2020.

*01)* Um título comprado por mil reais promete pagar juros reais de 10%
a.a. A alíquota de imposto é de 40%. Qual a inflação máxima no período para
que não hajam perdas reais?
Resp.: 17,62%

-- 

Pedro Jerônimo S. de O. Júnior

Professor de Matemática

-- 
Esta mensagem foi verificada pelo sistema de antiv�rus e
 acredita-se estar livre de perigo.



Re: [obm-l] Artigo

2021-04-02 Por tôpico Pedro Júnior
Boa discussão!

Em ter, 30 de mar de 2021 17:16, Israel Meireles Chrisostomo <
israelmchrisost...@gmail.com> escreveu:

> Obrigado
>
> Em ter., 30 de mar. de 2021 às 16:20, Daniel Jelin 
> escreveu:
>
>> não sei ao certo, meu caro, mas, falando como professor (e leitor),
>> suponho que não. e não é tanto por ser muito ou pouco avançado. receio que
>> o assunto fuja às preocupações do ensino básico - mesmo que a sua prova
>> seja elementar. repara, nada contra provas matemáticas na escola, ao
>> contrário. acho importante mostrar para os alunos de onde vêm os teoremas,
>> claro, mas: apenas das propriedades que eles de fato usam; e apenas as
>> demonstrações que eles têm condição de acompanhar do princípio ao fim, sem
>> que isso se torne um fardo adicional. será o caso?
>>
>> claro, algumas provas podem interessar ao professor mesmo que ele não
>> tenha a intenção de levá-la a todos os alunos. a irracionalidade de pi,
>> talvez, pra ficar no mesmo campo. leria com gosto uma investigação sobre a
>> irracionalidade de pi, passo a passo, com as armas da matemática do ensino
>> médio. tem isso? sei lá eu. mas a transcendência de pi? que tipo de
>> questão, que problema (escolar) esbarra na transcendência de pi? uma
>> sugestão: se vc puder mostrar que o professor deveria, sim, se importar com
>> isso, que questões importantes passam por aí, opa, então beleza, aí fica
>> mto legal, aí tem tudo a ver.
>>
>> abs
>>
>> On Tue, Mar 30, 2021 at 2:14 PM Israel Meireles Chrisostomo <
>> israelmchrisost...@gmail.com> wrote:
>>
>>> Vcs acham que a revista RPM aceitaria uma prova para transcendência de
>>> pi, ou isso é algo avançado demais para revista?
>>>
>>> --
>>> Israel Meireles Chrisostomo
>>>
>>> --
>>> Esta mensagem foi verificada pelo sistema de antivírus e
>>> acredita-se estar livre de perigo.
>>
>>
>> --
>> Esta mensagem foi verificada pelo sistema de antivírus e
>> acredita-se estar livre de perigo.
>
>
>
> --
> Israel Meireles Chrisostomo
>
> --
> Esta mensagem foi verificada pelo sistema de antivírus e
> acredita-se estar livre de perigo.

-- 
Esta mensagem foi verificada pelo sistema de antiv�rus e
 acredita-se estar livre de perigo.



[obm-l] Normas

2020-06-15 Por tôpico Pedro Júnior
[image: image.png]
Alguém pode me ajudar nesse problema?

-- 

Pedro Jerônimo S. de O. Júnior

Professor de Matemática

João Pessoa – PB

-- 
Esta mensagem foi verificada pelo sistema de antiv�rus e
 acredita-se estar livre de perigo.



Re: [obm-l] Artigos de Eureka

2018-09-26 Por tôpico Pedro Júnior
Obrigado, José! Mas é que eu estava procurando os artigos separados da
revista. Mas, como tem em word, fica até melhor.


Mais uma vez, obrigado!

Em ter, 25 de set de 2018 21:15, Pedro José  escreveu:

> Boa noite!
>
> Não só para PDF Creator, mas para Word também.
> Bons estudos!
>
>
> https://www.obm.org.br/revista-eureka/
>
> Saudações,
> PJMS
>
> Em ter, 25 de set de 2018 às 16:18, Pedro Júnior <
> pedromatematic...@gmail.com> escreveu:
>
>> Aqueles artigos de Eureka separados da revista em pdf tiraram do ar ou
>> estão em algum lugar? Não consigo encontrá-los. Se alguém tiver o link ou
>> souber de alguma coisa me avisa aqui. obrigado!
>>
>> --
>>
>> Pedro Jerônimo S. de O. Júnior
>>
>> Professor de Matemática
>>
>> João Pessoa – PB
>>
>> --
>> Esta mensagem foi verificada pelo sistema de antivírus e
>> acredita-se estar livre de perigo.
>
>
> --
> Esta mensagem foi verificada pelo sistema de antivírus e
> acredita-se estar livre de perigo.

-- 
Esta mensagem foi verificada pelo sistema de antiv�rus e
 acredita-se estar livre de perigo.



[obm-l] Artigos de Eureka

2018-09-25 Por tôpico Pedro Júnior
Aqueles artigos de Eureka separados da revista em pdf tiraram do ar ou
estão em algum lugar? Não consigo encontrá-los. Se alguém tiver o link ou
souber de alguma coisa me avisa aqui. obrigado!

-- 

Pedro Jerônimo S. de O. Júnior

Professor de Matemática

João Pessoa – PB

-- 
Esta mensagem foi verificada pelo sistema de antiv�rus e
 acredita-se estar livre de perigo.



[obm-l] Aritmética

2018-02-05 Por tôpico Pedro Júnior
Sejam m e n números naturais. Prove que

mn + 1 | 24 => m + n | 24.

Agradecido.

-- 

Pedro Jerônimo S. de O. Júnior

Professor de Matemática

Geo João Pessoa – PB

-- 
Esta mensagem foi verificada pelo sistema de antiv�rus e
 acredita-se estar livre de perigo.



[obm-l] Re: [obm-l] Convergência de Sequência - Ponto de Aderência

2017-10-30 Por tôpico Pedro Júnior
Obrigado a todos.

Em 30 de outubro de 2017 21:19, Artur Costa Steiner <steinerar...@gmail.com>
escreveu:

> Antes, veja o seguinte: se u é ponto de aderência da sequência (a_n),
> então (a_n) tem uma subsequência que converge para u.
>
> De fato, pela definição de ponto de aderência, para todo eps > 0 e todo M
> > 0 existe k > M tal que |a_k - u| < eps. Assim, existe k1 tal que |a_k_1-
> u| < 1. Suponhamos que, para algum n, existam k_1 < k_2 ...< k_n  tais que
> |a_k_i - u | < 1/i para i = 1, ... n. Pela definição de ponto de aderência,
> existe k_(n + 1) > k_n tal que |a_k_(n + 1)| < 1/(n + 1). Por indução
> concluímos que (a_k_n) é uma subsequência de (a_n) que converge para u,
> visto que 1/n —> 0.
>
> E pela definição de limite, vemos claramente que todo limite subsequencial
> de (a_n) é ponto de aderência de (a_n).
>
> Se (a_n) possuir dois pontos de aderência distintos, então (a_n) tem duas
> subsequências que convergem para limites distintos. Logo, (a_n) diverge.
> Assim, se a_n —> L, L é o único ponto de aderência de (a_n).
>
> Enviado do meu iPad
>
> Em 30 de out de 2017, à(s) 9:11 PM, Pedro Júnior <
> pedromatematic...@gmail.com> escreveu:
>
> Prove que uma sequência limitada converge para L, se, e somente se, L é
> o seu único ponto de aderência.
>
>
> Agradecido
> --
>
> Pedro Jerônimo S. de O. Júnior
>
> Professor de Matemática
>
> Geo João Pessoa – PB
>
> --
> Esta mensagem foi verificada pelo sistema de antivírus e
> acredita-se estar livre de perigo.
>
>
> --
> Esta mensagem foi verificada pelo sistema de antivírus e
> acredita-se estar livre de perigo.
>



-- 

Pedro Jerônimo S. de O. Júnior

Professor de Matemática

Geo João Pessoa – PB

-- 
Esta mensagem foi verificada pelo sistema de antiv�rus e
 acredita-se estar livre de perigo.



[obm-l] Convergência de Sequência - Ponto de Aderência

2017-10-30 Por tôpico Pedro Júnior
Prove que uma sequência limitada converge para L, se, e somente se, L é o
seu único ponto de aderência.


Agradecido
-- 

Pedro Jerônimo S. de O. Júnior

Professor de Matemática

Geo João Pessoa – PB

-- 
Esta mensagem foi verificada pelo sistema de antiv�rus e
 acredita-se estar livre de perigo.



[obm-l] Sequência

2017-10-30 Por tôpico Pedro Júnior
Mostre que $\sqrt {n^{2}+1} - \sqrt{n+h}$ tende a infinito

-- 

Pedro Jerônimo S. de O. Júnior

Professor de Matemática

Geo João Pessoa – PB

-- 
Esta mensagem foi verificada pelo sistema de antiv�rus e
 acredita-se estar livre de perigo.



[obm-l] Re: [obm-l] Re: [obm-l] Re: [obm-l] Lista preparatório para Olimpíada Universitária

2017-09-20 Por tôpico Pedro Júnior
Será que criar um grupo no whatsapp terá como consequência diminuir as
postagens de dúvidas aqui? Se não eu também quero participar do grupo.

Segue meu whatsapp 83 9 9893 5110

Desde já agradeço!

Em 20 de set de 2017 8:01 AM, "Matheus Fachini" 
escreveu:

Olá,

Também tenho interesse, meu número é (54)981296581.

Atenciosamente, Matheus.

Em ter, 19 de set de 2017 23:10, Daniel da Silva <
danielrochadasi...@icloud.com> escreveu:

> Boa noite,
>
> Também tenho interesse no grupo
> Nº (31) 98240-3789
>
> Obrigado,
> Daniel Rocha da Silva
>
> Em 19 de set de 2017, às 20:23, Leonardo Joau 
> escreveu:
>
> Boa noite,
>
> Igor no site poti.impa.br você consegue os materiais clicando em "baixar
> todo conteudo compactado".
> Tenho interesse em participar do grupo de treinento. Meu celular é 71
> 992920388.
>
>
> Abs,
> Leonardo
>
> --
> Esta mensagem foi verificada pelo sistema de antivírus e
> acredita-se estar livre de perigo.
>
>
> --
> Esta mensagem foi verificada pelo sistema de antivírus e
> acredita-se estar livre de perigo.
>

-- 
Esta mensagem foi verificada pelo sistema de antivírus e
acredita-se estar livre de perigo.

-- 
Esta mensagem foi verificada pelo sistema de antiv�rus e
 acredita-se estar livre de perigo.



Re: [obm-l] Dica Sobre Livros

2017-07-30 Por tôpico Pedro Júnior
Oi bom dia, gostaria do link dos livros.
Também quero ;)

Em 30 de jul de 2017 3:08 AM, "Lucas Colucci" 
escreveu:

Tenho interesse também. lucas.colucci.so...@gmail.com

Muito obrigado!

2017. júl. 30. 4:10 ezt írta ("Kelvin Anjos" ):

> Como essa lista é apenas para dúvidas e problemas da obm, te envio um
> e-mail com os anexos.
> Se alguém mais se interessar, basta me enviar um e-mail pedindo.
>
> On 29 July 2017 at 11:52, Ricardo Leão  wrote:
>
>> Eu tenho procurado os seguintes livros:
>>
>> - Andreescu, T; Kedlaya, K; Zeitz, P; *Mathematical Contests 1995-1996:
>> Olympiad Problems from around the world, with solutions* (1997)
>>
>> - Andreescu, T; Kedlaya, K; *Mathematical Contests 1996-1997: Olympiad
>> Problems from around the world, with solutions* (1998)
>>
>> - Andreescu, T; Kedlaya, K; *Mathematical Contests 1997-1998: Olympiad
>> Problems from around the world, with solutions* (1999)
>>
>> Alguém aí sabe onde eu encontro esses livros em formato físico ou
>> digital???
>>
>> --
>> Esta mensagem foi verificada pelo sistema de antivírus e
>> acredita-se estar livre de perigo.
>
>
>
> --
> Esta mensagem foi verificada pelo sistema de antivírus e
> acredita-se estar livre de perigo.


-- 
Esta mensagem foi verificada pelo sistema de antivírus e
acredita-se estar livre de perigo.

-- 
Esta mensagem foi verificada pelo sistema de antiv�rus e
 acredita-se estar livre de perigo.



[obm-l] Re: [obm-l] Re: [obm-l] Re: [obm-l] Re: [obm-l] Re: [obm-l] Polinômios

2017-07-25 Por tôpico Pedro Júnior
Obrigado, não havia percebido o deslize!

Em 25 de jul de 2017 10:48 PM, "Carlos Gomes" 
escreveu:


Pelo teorema do resto,

p(2)=p(3)=p(4)=r e p(1)=0

Considerando o polinômio q(x)=p(x)-r, segue que q(2)=q(3)=q(4)=0. Assim,

q(x)=A.(x-2)(x-3)(x-4), com A real. Portanto,

p(x)-r=q(x)=A.(x-2)(x-3)(x-4) ==> p(x)=A.(x-2)(x-3)(x-4)+r.

Ora, como p(1)=0, segue que 0=A(1-2)(1-3)(1-4)+r ==> r=6A

Assim, p(x)=A.(x-2)(x-3)(x-4)+¨6A

Variando o A nos reais (A não nulo) temos infinitos polinômios p cumprindo
as condições requeridas.

Cgomes.


-- 
Esta mensagem foi verificada pelo sistema de antivírus e
acredita-se estar livre de perigo.

-- 
Esta mensagem foi verificada pelo sistema de antiv�rus e
 acredita-se estar livre de perigo.



[obm-l] Re: [obm-l] Re: [obm-l] Polinômios

2017-07-25 Por tôpico Pedro Júnior
Obrigado, didático e criativo.
Valeu mesmo!

Em 25 de jul de 2017 9:55 PM, "Bruno Visnadi" <brunovisnadida...@gmail.com>
escreveu:

> Seja P(x) = (x-2)(x-3)(x-4) = x³ - 9x² + 26x - 24 -> P(1) = -6
>
> Seja Q(x) = P(x) + 6 -> Q(1) = 0 -> Q(x) é múltiplo de (x-1)
>
> Perceba que Q(x) deixa resto 6 por (x-2), (x-3) e (x-4). Todo polinômio no
> formato Q(x) + n*P(x), para todo n, deixa resto 6 por
> (x-2), (x-3) e (x-4).
>
> Em 25 de julho de 2017 21:22, Pedro Júnior <pedromatematic...@gmail.com>
> escreveu:
>
>> Prove que existem infinitos polinômios de grau 3 de coeficientes reais
>> que são divisíveis por x - 1 e que deixam o mesmo resto por x - 2, x - 3 e
>> x - 4.
>>
>> Quem tiver uma boa dica fica meus agradecimentos.
>>
>> --
>> Esta mensagem foi verificada pelo sistema de antivírus e
>> acredita-se estar livre de perigo.
>
>
>
> --
> Esta mensagem foi verificada pelo sistema de antivírus e
> acredita-se estar livre de perigo.

-- 
Esta mensagem foi verificada pelo sistema de antiv�rus e
 acredita-se estar livre de perigo.



[obm-l] Polinômios

2017-07-25 Por tôpico Pedro Júnior
Prove que existem infinitos polinômios de grau 3 de coeficientes reais que
são divisíveis por x - 1 e que deixam o mesmo resto por x - 2, x - 3 e x -
4.

Quem tiver uma boa dica fica meus agradecimentos.

-- 
Esta mensagem foi verificada pelo sistema de antiv�rus e
 acredita-se estar livre de perigo.



Re: [obm-l] Radicais

2017-06-04 Por tôpico Pedro Júnior
Show de bola!
Quanta criatividade...
Parabéns

Em 4 de jun de 2017 4:56 PM, "Esdras Muniz" <esdrasmunizm...@gmail.com>
escreveu:

> Comece com a identidade: $n^2\,=\,1+(n-1)\sqrt{(n+1)^2}$.
>
> $3=\sqrt{3^2}=\sqrt{1+2\sqrt{4^2}}=\sqrt{1+2\sqrt{1+3\sqrt{5^2}}}=\cdots$
>
> Em 4 de junho de 2017 15:40, Douglas Oliveira de Lima <
> profdouglaso.del...@gmail.com> escreveu:
>
>> Opa amigo, o radical do Indiano Ramanujam, baixe um arquivo do Carlos
>> Victor , muito bom tem esse problema resolvido e vários outros.
>> Segue o link http://cursos.ufrrj.br/posgraduacao/profmat/dissertacoe
>> s/dissertacoe/
>>
>> Um abraço
>> Douglas Oliveira.
>>
>> Em 4 de jun de 2017 3:19 PM, "Pedro Júnior" <pedromatematic...@gmail.com>
>> escreveu:
>>
>>> Olá pessoal, vocês poderiam me ajudar a solucionar o problema abaixo? Já
>>> vi alguns bem parecidos, mas esse está me pegando...
>>>
>>> Raiz (1+2Raiz(1+3Raiz(1+4Raiz(1+...= ?
>>>
>>> Desde já agradeço
>>>
>>> --
>>> Esta mensagem foi verificada pelo sistema de antivírus e
>>> acredita-se estar livre de perigo.
>>
>>
>> --
>> Esta mensagem foi verificada pelo sistema de antivírus e
>> acredita-se estar livre de perigo.
>>
>
>
>
> --
> Esdras Muniz Mota
> Mestrando em Matemática
> Universidade Federal do Ceará
>
>
>
> --
> Esta mensagem foi verificada pelo sistema de antivírus e
> acredita-se estar livre de perigo.

-- 
Esta mensagem foi verificada pelo sistema de antiv�rus e
 acredita-se estar livre de perigo.



[obm-l] Radicais

2017-06-04 Por tôpico Pedro Júnior
Olá pessoal, vocês poderiam me ajudar a solucionar o problema abaixo? Já vi
alguns bem parecidos, mas esse está me pegando...

Raiz (1+2Raiz(1+3Raiz(1+4Raiz(1+...= ?

Desde já agradeço

-- 
Esta mensagem foi verificada pelo sistema de antiv�rus e
 acredita-se estar livre de perigo.



[obm-l] Re: [obm-l] Re: [obm-l] Sequências

2016-11-16 Por tôpico Pedro Júnior
É da forma 4x. Logo A_1, A_2, A_3, ..., A_n a soma de seus elementos é um
múltiplo de 4, logo múltiplo de 2, ou seja, par.
Ou seja, 4n^{2} + n tem que ser par, logo, n é par. E a segunda parte do
problema Ralph?

Em 16 de novembro de 2016 22:09, Ralph Teixeira <ralp...@gmail.com>
escreveu:

> Dica para comecar: se A_k={a,b,c,x} onde x eh a media de a,b e c, o que
> voce pode dizer sobre a soma dos elementos de A_k?
>
> Abraco, Ralph.
>
> 2016-11-16 21:58 GMT-02:00 Pedro Júnior <pedromatematic...@gmail.com>:
>
>> Ainda não consegui esse problema. Ele foi do livro do Caminha.
>> Ache todos os valores de $n$ para os quais possamos escrever o conjunto
>> A={1,2,3,..., 4n} como união de n conjuntos, dois a dois disjuntos e com
>> 4 elementos cada, tais que em cada um deles um dos elementos seja igual à
>> média aritmética dos três demais (sugestão: suponha inicialmente $ A= A_{1}
>> \cup \ldots \cup A_{n} $ com $ A_{1}, \ldots, A_{n} $ satisfazendo as
>> condições do enunciado, e conclua daí que $n$ deve ser par. Em seguida,
>> mostre - exibindo uma maneira de escrever $A$ como pedido - que para todo
>> $n$ par serve).
>>
>> --
>>
>> Pedro Jerônimo S. de O. Júnior
>>
>> Professor de Matemática
>>
>> Geo João Pessoa – PB
>>
>> --
>> Esta mensagem foi verificada pelo sistema de antivírus e
>> acredita-se estar livre de perigo.
>
>
>
> --
> Esta mensagem foi verificada pelo sistema de antivírus e
> acredita-se estar livre de perigo.




-- 

Pedro Jerônimo S. de O. Júnior

Professor de Matemática

Geo João Pessoa – PB

-- 
Esta mensagem foi verificada pelo sistema de antiv�rus e
 acredita-se estar livre de perigo.



[obm-l] Sequências

2016-11-16 Por tôpico Pedro Júnior
Ainda não consegui esse problema. Ele foi do livro do Caminha.
Ache todos os valores de $n$ para os quais possamos escrever o conjunto
A={1,2,3,..., 4n} como união de n conjuntos, dois a dois disjuntos e com 4
elementos cada, tais que em cada um deles um dos elementos seja igual à
média aritmética dos três demais (sugestão: suponha inicialmente $ A= A_{1}
\cup \ldots \cup A_{n} $ com $ A_{1}, \ldots, A_{n} $ satisfazendo as
condições do enunciado, e conclua daí que $n$ deve ser par. Em seguida,
mostre - exibindo uma maneira de escrever $A$ como pedido - que para todo
$n$ par serve).

-- 

Pedro Jerônimo S. de O. Júnior

Professor de Matemática

Geo João Pessoa – PB

-- 
Esta mensagem foi verificada pelo sistema de antiv�rus e
 acredita-se estar livre de perigo.



[obm-l] Aritmética

2016-03-30 Por tôpico Pedro Júnior
Qual o maior resto possível da divisão de um número de dois algarismos pela
soma de seus algarismos?
Achei que no Abrano Hefez havia algo relacionado com isso, mas não
encontrei.
Caso saibam de alguma fórmula ou teoria gostaria do link ou referência.
Obrigado

-- 

Pedro Jerônimo S. de O. Júnior

Professor de Matemática

Geo João Pessoa – PB

-- 
Esta mensagem foi verificada pelo sistema de antiv�rus e
 acredita-se estar livre de perigo.



[obm-l] Re: [obm-l] Re: [obm-l] Existência de Função

2016-03-22 Por tôpico Pedro Júnior
Esqueci de dizer que X e Y são infinitos.
E então, como mostro que existe.
Em 22 de mar de 2016 7:31 AM, "Bernardo Freitas Paulo da Costa" <
bernardo...@gmail.com> escreveu:

> 2016-03-22 5:11 GMT-03:00 Pedro Júnior <pedromatematic...@gmail.com>:
> > Se f: X --> Y é injetiva e g: N --> Y é bijetiva, mostre que existe h: N
> -->
> > X bijetiva.
> >
> > obs.: N:= naturais
>
> Isso é falso. Tome X = {1}, Y = N. f(1) = 1 é injetiva (toda função de
> um conjunto com um único elemento é injetiva!). g é a identidade, que
> é obviamente bijetiva. Não existe h : N -> X porque X não é infinito.
>
> Se você pedir que X seja infinito, então é verdade, porque "N é o
> menor infinito", e a hipótese é equivalente a existir f2 : X -> N
> injetiva.
>
> Abraços,
> --
> Bernardo Freitas Paulo da Costa
>
> --
> Esta mensagem foi verificada pelo sistema de antivírus e
>  acredita-se estar livre de perigo.
>
>
> =
> Instru�ões para entrar na lista, sair da lista e usar a lista em
> http://www.mat.puc-rio.br/~obmlistas/obm-l.html
> =
>

-- 
Esta mensagem foi verificada pelo sistema de antiv�rus e
 acredita-se estar livre de perigo.



[obm-l] Existência de Função

2016-03-22 Por tôpico Pedro Júnior
Se f: X --> Y é injetiva e g: N --> Y é bijetiva, mostre que existe h: N
--> X bijetiva.

obs.: N:= naturais

-- 

Pedro Jerônimo S. de O. Júnior

Professor de Matemática

Geo João Pessoa – PB

-- 
Esta mensagem foi verificada pelo sistema de antiv�rus e
 acredita-se estar livre de perigo.



[obm-l] Limite por épsilon e delta

2014-05-02 Por tôpico Pedro Júnior
Calcular, por épsilon e delta, o limite da sequência: x_n = (sen n ) / (n²
- n).

-- 

Pedro Jerônimo S. de O. Júnior

Professor de Matemática

Geo João Pessoa – PB

-- 
Esta mensagem foi verificada pelo sistema de antiv�rus e
 acredita-se estar livre de perigo.



[obm-l] Re: [obm-l] Re: [obm-l] Limite por épsilon e delta

2014-05-02 Por tôpico Pedro Júnior
Certo, e como faz?


Em 2 de maio de 2014 21:24, Pacini Bores pacini.bo...@globo.com escreveu:

 Olá  Pedro,

 Em geral avalio que a pergunta deveria ser :

 1) Calcule o Limite da sequência, caso exista.

 2) Depois, mostre que o limite é o valor calculado em (1), utilizando a
 definição de limite de uma sequência.

 Pacini


 Em 2 de maio de 2014 19:48, Pedro Júnior 
 pedromatematic...@gmail.comescreveu:

 Calcular, por épsilon e delta, o limite da sequência: x_n = (sen n ) /
 (n² - n).

 --

 Pedro Jerônimo S. de O. Júnior

 Professor de Matemática

 Geo João Pessoa – PB

 --
 Esta mensagem foi verificada pelo sistema de antivírus e
 acredita-se estar livre de perigo.



 --
 Esta mensagem foi verificada pelo sistema de antivírus e
 acredita-se estar livre de perigo.




-- 

Pedro Jerônimo S. de O. Júnior

Professor de Matemática

Geo João Pessoa – PB

-- 
Esta mensagem foi verificada pelo sistema de antiv�rus e
 acredita-se estar livre de perigo.



[obm-l] Aritmética

2013-12-04 Por tôpico Pedro Júnior
Olá pessoal gostaria de uma ajuda na reolução do problema:
1) Mostre que existem infinitos valores de n (natural) para os quais 8n^2 +
5 ẽ divisível por 77.

Desde já agradeço

-- 

Pedro Jerônimo S. de O. Júnior

Professor de Matemática

Geo João Pessoa – PB

-- 
Esta mensagem foi verificada pelo sistema de antiv�rus e
 acredita-se estar livre de perigo.



[obm-l] Re: [obm-l] Re: [obm-l] Aritmética

2013-12-04 Por tôpico Pedro Júnior
Obrigado Cássio, mas não pensei que fosse tão complicado! (pelo menos pra
mim!)
Caso alguém consiga de uma forma diferente favor encaminhar.
Abç
Pedro Jr


Em 4 de dezembro de 2013 13:50, Cassio Anderson Feitosa 
cassiofeito...@gmail.com escreveu:

 8n^2+5\equiv 0\pmod 77  é equivalente a 8n^2+5 == 0 mod 7e8n^2+5==
 0 mod 11.

 Primeira parte: 8n² == 5 mod 11  == 8n^2 == 6mod 11 == 4n² == 3 mod 11
 == 3(4n²) == 9 mod 11 ==  12n²==n²==9 mod 11 ===n==3 ou n== -3 mod
 11, ou seja, n==3 ou n== 8 mod 11.

  Segunda parte: 8n² == 5 mod 7 == 8n^2 == 2mod 7 == 4n² == 1 mod 7 ==
 2(4n²) == 2 mod 7 ==  8n²==n²==2 mod 7. =   n==3 ou n== -3 mod 11,
 ou seja, n==3 ou n== 4 mod 7.

  Então, o sistema n == 3 mod 11 e n == 3 mod 7 gera uma solução.
  o sistema  n == 3 mod 11 e n == 4 mod 7 gera outra solução
 n == 8 mod 11 e n == 3 mod 7 outra solução
 n == 8 mod 11 e n == 4 mod 7 outra solução.


  Daí basta pegar cada sistema de duas congruências e resolver pelo Teorema
 chinês de Resto.

  Por exemplo, a solução pro primeiro sistema é n=77q + 3, q inteiro.




 --
 Cássio Anderson
 Graduando em Matemática - UFPB

 --
 Esta mensagem foi verificada pelo sistema de antivírus e
 acredita-se estar livre de perigo.




-- 

Pedro Jerônimo S. de O. Júnior

Professor de Matemática

Geo João Pessoa – PB

-- 
Esta mensagem foi verificada pelo sistema de antivírus e
 acredita-se estar livre de perigo.



Re: [obm-l] MDC

2013-09-29 Por tôpico Pedro Júnior
Sim, sim obrigado!


Em 28 de setembro de 2013 21:47, terence thirteen
peterdirich...@gmail.comescreveu:



 Em 28 de setembro de 2013 15:56, Pedro Júnior pedromatematic...@gmail.com
  escreveu:

 Como mostro que mdc(an,bn)=n. mdc(a,b).

 A proposição é claríssima, mas não estou conseguindo concluir.


 Vamos pelo velho método indígena: fatoração!

 Por demonstração, o MDC de dois caras consiste no produto de todos os
 primos comuns a ambos os termos, cada um deles com o menor expoente que
 estiver presente entre ambos. Por exemplo, MDC (2^3*3^2, 2^5*5^1) = 2^3

 Assim sendo, vamos pegar um primo p. este primo aparece A vezes em a, B
 vezes em b, N vezes em n.

 Então, ele aparecerá A+N vezes em an, B+N vezes em bn, e portanto
 aparecerá MAX(A+N,B+N) vezes em MDC(an,bn)
 Igualmente, ele aparecerá A vezes em a, B vezes em b, e portanto aparecerá
 N+MAX(A,B) vezes em n*MDC(a,b)

 Assim, basta demonstrar que MAX(A+N,B+N) = N+MAX(A,B).

 Me parece mais fácil, não? :P



 Uma dica para o futuro: as funções MDC e MMC são duais a MAX e MIN.




 --

 Pedro Jerônimo S. de O. Júnior

 Geo João Pessoa – PB

 --
 Esta mensagem foi verificada pelo sistema de antivírus e
 acredita-se estar livre de perigo.




 --
 /**/
 神が祝福

 Torres

 --
 Esta mensagem foi verificada pelo sistema de antivírus e
 acredita-se estar livre de perigo.




-- 

Pedro Jerônimo S. de O. Júnior

Professor de Matemática

Geo João Pessoa – PB

-- 
Esta mensagem foi verificada pelo sistema de antiv�rus e
 acredita-se estar livre de perigo.



[obm-l] MDC

2013-09-28 Por tôpico Pedro Júnior
Como mostro que mdc(an,bn)=n. mdc(a,b).

A proposição é claríssima, mas não estou conseguindo concluir.

-- 

Pedro Jerônimo S. de O. Júnior

Geo João Pessoa – PB

-- 
Esta mensagem foi verificada pelo sistema de antivírus e
 acredita-se estar livre de perigo.



[obm-l] Combinatória - Bandeira

2013-07-14 Por tôpico Pedro Júnior
Considere a bandeira da figura abaixo, formada por seis regiões. Para
colori-la, há
lápis de cor de quatro cores diferentes.
[image: Imagem inline 1]
a)  De quantos modos ela pode ser colorida de modo que regiões adjacentes
tenham cores diferentes?
b) Resolva o item a), supondo agora que todas as quatro cores sejam
utilizadas para pintar cada bandeira.

Como resolver a letra (b) de forma direta?

-- 

Pedro Jerônimo S. de O. Júnior

Professor de Matemática

Geo João Pessoa – PB

-- 
Esta mensagem foi verificada pelo sistema de antivírus e
 acredita-se estar livre de perigo.

bandeira.png

[obm-l] Probabiliedade

2013-07-08 Por tôpico Pedro Júnior
1. Quatro times, entre os quais o JUBA, disputam um torneio de vôlei em
que:

   - Cada time joga contra cada um dos outros uma única vez;
   - Qualquer partida termina com a vitória de um dos times;
   - Em qualquer partida, os times têm a mesma probabilidade de ganhar;
   - Ao final do torneio, os times são classificados em ordem de vitórias.

a)  É possível que, ao final do torneio, todos os times tenham o mesmo
número de vitórias? Por quê?

b)  Qual é a probabilidade de que o torneio termine com o JUBA isolado
em primeiro lugar?

c)   Qual é a probabilidade de que o torneio termine com três times
empatados em primeiro lugar?

-- 
Esta mensagem foi verificada pelo sistema de antivírus e
 acredita-se estar livre de perigo.



[obm-l] Função Quadrática e Desigualdade

2013-04-07 Por tôpico Pedro Júnior
Seja f(x) = ax² + bx + c com a  0. Mostre que f((x+y)/2)  [f(x) +f(y)]/2.

-- 

Pedro Jerônimo S. de O. Júnior

Professor de Matemática

Geo João Pessoa – PB

-- 
Esta mensagem foi verificada pelo sistema de antivírus e
 acredita-se estar livre de perigo.



[obm-l] Re: [obm-l] RE: [obm-l] Função Quadrática e Desigualdade

2013-04-07 Por tôpico Pedro Júnior
Falou João, muito obrigado!


Em 7 de abril de 2013 15:16, João Maldonado
joao_maldona...@hotmail.comescreveu:

 É o teorema de Jensen, temos que provar que a função é convexa (meio fácil
 de ver né? )
 Suponha o contrário, ou seja,
 f((x+y)/2) = [f(x) +f(y)]/2.
 E suponha x!=y


 teríamos
 a(x+y)²/4 + b(x+y)/2 + c = a(x²+y²)/2 + b(x+y)/2 + c  =
 (x+y)² = 2(x²+y²)
 (x-y)²=0, absurdo

 []'s
 João


 Date: Sun, 7 Apr 2013 13:43:42 -0300
 Subject: [obm-l] Função Quadrática e Desigualdade
 From: pedromatematic...@gmail.com
 To: obm-l@mat.puc-rio.br


 Seja f(x) = ax² + bx + c com a  0. Mostre que f((x+y)/2)  [f(x) +f(y)]/2.

 --
 Pedro Jerônimo S. de O. Júnior
 Professor de Matemática
 Geo João Pessoa – PB

 --
 Esta mensagem foi verificada pelo sistema de antivírus e
 acredita-se estar livre de perigo.

 --
 Esta mensagem foi verificada pelo sistema de antivírus e
 acredita-se estar livre de perigo.




-- 

Pedro Jerônimo S. de O. Júnior

Professor de Matemática

Geo João Pessoa – PB

-- 
Esta mensagem foi verificada pelo sistema de antivírus e
 acredita-se estar livre de perigo.



[obm-l] Re: [obm-l] RE: [obm-l] Função Quadrática e Desigualdade

2013-04-07 Por tôpico Pedro Júnior
Já vi que usando o teorema fica simples... mas fiquei curioso com uma
coisa: dos arquivos que baixei sobre a desiguldade de Jansen, nenhum deles
mostra como foi intuida tal desigualdade. Usam indução numa desigualdade
que surgiu de onde? Será que te uma prova direta... ou só o fato geométrico
é suficiente para intuir tal desigualdade?


2013/4/7 Hyon Ferreira Cordeiro h-y-o...@hotmail.com

 Temos que f''(x)= 2a 0 para todo x.
 Segue de Jensen que f(x+y/2)  (f(x)+f(y))/2

 --
 Date: Sun, 7 Apr 2013 13:43:42 -0300
 Subject: [obm-l] Função Quadrática e Desigualdade
 From: pedromatematic...@gmail.com
 To: obm-l@mat.puc-rio.br

 Seja f(x) = ax² + bx + c com a  0. Mostre que f((x+y)/2)  [f(x) +f(y)]/2.

 --
 Pedro Jerônimo S. de O. Júnior
 Professor de Matemática
 Geo João Pessoa – PB

 --
 Esta mensagem foi verificada pelo sistema de antivírus e
 acredita-se estar livre de perigo.

 --
 Esta mensagem foi verificada pelo sistema de antivírus e
 acredita-se estar livre de perigo.




-- 

Pedro Jerônimo S. de O. Júnior

Professor de Matemática

Geo João Pessoa – PB

-- 
Esta mensagem foi verificada pelo sistema de antivírus e
 acredita-se estar livre de perigo.



[obm-l] Re: [obm-l] RE: [obm-l] Equação Funcional ou Recorrência

2013-03-31 Por tôpico Pedro Júnior
Olá Leandro, consegui resolver o problema  e muito obrigado pela sugestão.
Seguinte:


Faça x = 0  == f(f(0)) + af(0) = b(a+b) 0 = 0 == f(f(0)) = - af(0)

Seja f(0) = y_1 == f(f(0)) = f(y_1) = - a f(0)
Agora faça f(y_1) = y_2

perceba a recorrência: y_n = f(y_(n-1)). Substituindo na equação funcional
temos:

y_(n+2) +a y_(n+1) -b(a+b) y_n = 0

Eq. Característica: r^2 +ar - b(a+b) = 0

Daí continua...
Abç


Em 31 de março de 2013 16:48, LEANDRO L RECOVA leandrorec...@msn.comescreveu:

 Eu pensei no seguinte:

 y=f(x). Entao,

 f(y) + ay = b(a+b)x

 f(y) = b(a+b)x-ay

 Ja que f assume valores reais positivos (R^+), entao, temos que ter f(y) 
 0, ou seja,

 ay  b(a+b)x  = f(x)  b/a (a+b)x.  (*)

 As funcoes f devem satisfazer a condicao (*).  Vou continuar pensando na
 questao.

 --
 Date: Sat, 30 Mar 2013 16:09:29 -0300
 Subject: [obm-l] Equação Funcional ou Recorrência
 From: pedromatematic...@gmail.com
 To: obm-l@mat.puc-rio.br


 Essa questão estava numa lista de Equações Recorrentes. É possível
 resolvê-la por recorrência?

 Ache todas as funções f: R^+ --R^+ tais que f(f(x)) + af(x) = b(a+b)x
 onde a,b \in R^+.

 --
 Pedro Jerônimo S. de O. Júnior
 Professor de Matemática
 Geo João Pessoa – PB

 --
 Esta mensagem foi verificada pelo sistema de antivírus e
 acredita-se estar livre de perigo.

 --
 Esta mensagem foi verificada pelo sistema de antivírus e
 acredita-se estar livre de perigo.




-- 

Pedro Jerônimo S. de O. Júnior

Professor de Matemática

Geo João Pessoa – PB

-- 
Esta mensagem foi verificada pelo sistema de antivírus e
 acredita-se estar livre de perigo.



[obm-l] Re: [obm-l] Re: [obm-l] RE: [obm-l] Contagem difícil

2012-09-30 Por tôpico Pedro Júnior
Olá Bernardo, você tem esse livro em pdf ou djavu?
Ou sabes onde está postado para download?

Aradeço antecipadamente.

Pedro Jr

Em 27 de setembro de 2012 22:31, Bernardo Freitas Paulo da Costa 
bernardo...@gmail.com escreveu:

 2012/9/27 Athos Couto athos...@hotmail.com:
  Boa noite.
  Eu ainda estou no ensino médio, mas já tive um contato com funções
 geradoras
  quando dei uma lida sobre convoluções.
  Eu não consegui acompanhar a resolução, não faz sentido para mim, hehe,
  falta teoria.
  Qual o nome dessa matéria? Será que tem algum material online para eu dar
  uma olhada?
 Tem um livro que é referência para várias técnicas de contagem:
 Matemática discreta, de Graham, Knuth, Patashnik. Fala de um monte de
 técnicas diferentes, inclusive séries geratrizes e outras mágicas a
 mais. E é muito bom de ler, os caras são muito didáticos.

 Abraços,
 --
 Bernardo Freitas Paulo da Costa

 =
 Instruções para entrar na lista, sair da lista e usar a lista em
 http://www.mat.puc-rio.br/~obmlistas/obm-l.html
 =




-- 

Pedro Jerônimo S. de O. Júnior

Professor de Matemática

Geo João Pessoa – PB


[obm-l] Probabilidade

2012-05-07 Por tôpico Pedro Júnior
Se lançarmos diversas vezes dois dados, um vermelho e um branco,
e cacularmos a diferença entre os pontos obtidos, quais as diferenças
mais frequêntes?

-- 

Pedro Jerônimo S. de O. Júnior

Professor de Matemática

Geo João Pessoa – PB


[obm-l] Re: [obm-l] Re: [obm-l] indução

2012-04-07 Por tôpico Pedro Júnior
Isso mostra a questão colocada pelo Maldonado...

Em 7 de abril de 2012 11:32, Alex pereira Bezerra 
alexmatematica1...@gmail.com escreveu:

 [image:
 \displaystyle\frac{1}{2}\cdot\frac{3}{4}\cdots\frac{2n-1}{2n}\leq\frac{1}{\sqrt{3n+1}}.]http://moodle.profmat-sbm.org.br/filter/tex/displaytex.php?texexp=%5Cdisplaystyle%5Cfrac%7B1%7D%7B2%7D%5Ccdot%5Cfrac%7B3%7D%7B4%7D%5Ccdots%5Cfrac%7B2n-1%7D%7B2n%7D%5Cleq%5Cfrac%7B1%7D%7B%5Csqrt%7B3n%2B1%7D%7D.

 Prova-se usando indução. É claro que a desigualdade é válida para n=1.
 Supondo válida para n maior ou igual a 1, devemos mostrar que também vale
 para (n+1), ou seja, mostrar que:

 [image:
 \displaystyle\frac{1}{2}\cdot\frac{3}{4}\cdots\frac{2n-1}{2n}\cdot\frac{2n+1}{2n+2}\leq\frac{1}{\sqrt{3n+4}}.]http://moodle.profmat-sbm.org.br/filter/tex/displaytex.php?texexp=%5Cdisplaystyle%5Cfrac%7B1%7D%7B2%7D%5Ccdot%5Cfrac%7B3%7D%7B4%7D%5Ccdots%5Cfrac%7B2n-1%7D%7B2n%7D%5Ccdot%5Cfrac%7B2n%2B1%7D%7B2n%2B2%7D%5Cleq%5Cfrac%7B1%7D%7B%5Csqrt%7B3n%2B4%7D%7D.

 Mas, por hipótese

 [image:
 \displaystyle\left(\frac{1}{2}\cdot\frac{3}{4}\cdots\frac{2n-1}{2n}\right)
 \cdot\frac{2n+1}{2n+2}\leq\frac{1}{\sqrt{3n+1}}\cdot\frac{2n+1}{2n+2}.]http://moodle.profmat-sbm.org.br/filter/tex/displaytex.php?texexp=%5Cdisplaystyle%5Cleft%28%5Cfrac%7B1%7D%7B2%7D%5Ccdot%5Cfrac%7B3%7D%7B4%7D%5Ccdots%5Cfrac%7B2n-1%7D%7B2n%7D%5Cright%29%20%5Ccdot%5Cfrac%7B2n%2B1%7D%7B2n%2B2%7D%5Cleq%5Cfrac%7B1%7D%7B%5Csqrt%7B3n%2B1%7D%7D%5Ccdot%5Cfrac%7B2n%2B1%7D%7B2n%2B2%7D.

 Mostremos então que

 [image:
 \displaystyle\frac{1}{\sqrt{3n+1}}\cdot\frac{2n+1}{2n+2}\leq\frac{1}{\sqrt{3n+4}}.]http://moodle.profmat-sbm.org.br/filter/tex/displaytex.php?texexp=%5Cdisplaystyle%5Cfrac%7B1%7D%7B%5Csqrt%7B3n%2B1%7D%7D%5Ccdot%5Cfrac%7B2n%2B1%7D%7B2n%2B2%7D%5Cleq%5Cfrac%7B1%7D%7B%5Csqrt%7B3n%2B4%7D%7D.

 Como se tratam de números positivos, provar esta desigualdade é
 equivalente a provar a desigualdade para seus quadrados pois

 [image: 0 x,\,y\,\,\, ent\~ao\,\,\, x\leq y \,\,\Leftrightarrow\,\,
 x^2\leq 
 y^2.]http://moodle.profmat-sbm.org.br/filter/tex/displaytex.php?texexp=0%3C%20x%2C%5C%2Cy%5C%2C%5C%2C%5C%2C%20ent%5C%7Eao%5C%2C%5C%2C%5C%2C%20x%5Cleq%20y%20%5C%2C%5C%2C%5CLeftrightarrow%5C%2C%5C%2C%20x%5E2%5Cleq%20y%5E2.

 Temos

 [image: 
 (3n+1)(2n+2)^2=12n^3+28n^2+20n+4=(3n+4)(2n+1)^2+n]http://moodle.profmat-sbm.org.br/filter/tex/displaytex.php?texexp=%283n%2B1%29%282n%2B2%29%5E2%3D12n%5E3%2B28n%5E2%2B20n%2B4%3D%283n%2B4%29%282n%2B1%29%5E2%2Bn

 [image: 
 \geq(3n+4)(2n+1)^2.]http://moodle.profmat-sbm.org.br/filter/tex/displaytex.php?texexp=%5Cgeq%283n%2B4%29%282n%2B1%29%5E2.

 Logo,

 [image:
 \displaystyle\frac{1}{(3n+1)}\cdot\frac{(2n+1)^2}{(2n+2)^2}\leq\frac{1}{(3n+4)}]http://moodle.profmat-sbm.org.br/filter/tex/displaytex.php?texexp=%5Cdisplaystyle%5Cfrac%7B1%7D%7B%283n%2B1%29%7D%5Ccdot%5Cfrac%7B%282n%2B1%29%5E2%7D%7B%282n%2B2%29%5E2%7D%5Cleq%5Cfrac%7B1%7D%7B%283n%2B4%29%7D

 o que mostra que a desigualdade também vale para (n+1). Pelo Princípio de
 Indução segue que vale para todo número natural.


 Em 6 de abril de 2012 09:33, marcone augusto araújo borges 
 marconeborge...@hotmail.com escreveu:

  Alguem poderia me ajudar nessa questão?

 Provar por indução que 1/2*3/4*5/6...*(2n-1)/2n  = 1/raiz(3n+1),para
 todo n natural.





-- 

Pedro Jerônimo S. de O. Júnior

Professor de Matemática

Geo João Pessoa – PB


Re: [obm-l] (EN) Contagem

2012-02-25 Por tôpico Pedro Júnior
olhe que é 22

Em 24 de fevereiro de 2012 18:17, Carlos Nehab carlos.ne...@gmail.comescreveu:

 Gostei.
 Bem mais simples que a minha sugestão.
 Abraços
 Nehab

 Em 24/02/2012 00:33, terence thirteen escreveu:

 Poxa, gente, é mais fácil que isso!
 Todos os números que só tem 2,3,4,5,6? Simples: associe cada um delescom
 aquele que seja feito trocando os dígitos por aqueles que faltampra somar 8.
 Enfim, troque 2 com 6, 3 com 5 e 4 com 4.
 Exemplo: 24365--  64523. É impossível que dẽ dois caras iguais - sóse
 fosse tudo 4, mas só vale repetidos.
 Assim, a soma desses pares é 8. Multiplica pelo tanto de pares e fim!
 Como sempre, Gauss somando de 1 até 100 na raça...
 Em 23 de fevereiro de 2012 22:06, Carlos Nehabcarlos.ne...@gmail.com
  escreveu:  Poxa, Arkon,  No braço? Se poupe...  Imagine que você esteja
 somando todos o números na vertical. As unidades  primeiro: quantas vezes
 você somará algarismos 2, algarismos 3, etc?  Abraços,  Nehab  Em
 23/02/2012 20:40, arkon escreveu:  Pessoal, qual o bizu ou é só no
 braço mesmo???  Com os algarismos 2, 3, 4, 5 e 6 formam-se todos os
 números de 5  algarismos distintos.  Determine a soma de todos eles.
  
 ==**==**=
  Instru��es para entrar na lista, sair da lista e usar a lista em
 http://www.mat.puc-rio.br/~**obmlistas/obm-l.htmlhttp://www.mat.puc-rio.br/%7Eobmlistas/obm-l.html
  
 ==**==**=
  
 ==**==**=
  Instru�ões para entrar na lista, sair da lista e usar a lista em
 http://www.mat.puc-rio.br/~**obmlistas/obhttp://www.mat.puc-rio.br/%7Eobmlistas/ob
 !

 m-l.html  ===!

 ==**==**==


 -- //神が祝福
 Torres
 ==**==**
 =
 Instru��es para entrar na lista, sair da lista e usar a lista em
 http://www.mat.puc-rio.br/~**obmlistas/obm-l.htmlhttp://www.mat.puc-rio.br/%7Eobmlistas/obm-l.html
 ==**==**
 =




 ==**==**
 =
 Instru�ões para entrar na lista, sair da lista e usar a lista em
 http://www.mat.puc-rio.br/~**obmlistas/obm-l.htmlhttp://www.mat.puc-rio.br/%7Eobmlistas/obm-l.html
 ==**==**
 =




-- 

Pedro Jerônimo S. de O. Júnior

Professor de Matemática

Geo João Pessoa – PB


Re: [obm-l] Mais 4 - Treinamento IME

2011-10-22 Por tôpico Pedro Júnior
Questão 03)
Desenvolvendo o produto (a+b)(a+c) = a^2+ac+ab+bc = a(a+b+c) +bc agora
exatamente nesta soma use a desigualdade MA = MG, ficando assim:
[a(a+b+c) +bc]/2 = sqrt[a(a+b+c) +bc] = sqrt[abc(a+b+c)] == a(a+b+c) +bc
=2 sqrt[abc(a+b+c)] .
Pronto, acabou!!!

2011/10/21 João Maldonado joao_maldona...@hotmail.com

  Olá a toodos

 Primeiramente obrigado  ao Douglas, Terence e Luan  que me ajudaram nos
  outros 2 problemas
 Particularmente a resolução do Luan  da equação me ajudou tambm no nono
 problema da quarta proova

 Ainda restam 4 problemas que não consegui resolver  das 4 provas de
 treinamento.

 Se alguém puder me ajudar eu agradeço

 Prova 3  -   http://www.rumoaoita.com/site/attachments/044_simulado_3.pdf


 8) Se  a/(b+c) + b/(a+c)  + c/(a+b) = 1. Calcule a²/(b+c) + b²/(a+c)  +
 c²/(a+b)

 9) Determine o valor da soma  C(0, n) + C(1, n).sen(k) + C(2, n).sen (2k) +
 ... + C(n, n).sen(nk)

 Prova 4 - http://www.rumoaoita.com/site/attachments/044_simulado_4.pdf

 3) Prove  que (a+b)(a+c) =  2(abc(a+b+c))^(1/2),  para quaisquer positivos
 a, b, c

 5)  Parte 2
 Calcule a soma  das soluções da equação
 cossec(13x) + sen(13x) =  2cos(3x), com  0=x = Pi/2

 []'s
 João




-- 

Pedro Jerônimo S. de O. Júnior

Professor de Matemática

Geo João Pessoa – PB


[obm-l] Função de Euler - T. Números

2011-09-26 Por tôpico Pedro Júnior
Alguém sabe uma demonstração bem legal para a propriedade phi(x.y) =
phi(x) . phi(y), onde essa função é a phi de Euler?

-- 

Pedro Jerônimo S. de O. Júnior

Professor de Matemática

Geo João Pessoa – PB


[obm-l] Essa ainda não consegui!!!

2011-07-24 Por tôpico Pedro Júnior
Sejam A, B, C, D e E pontos do plano cartesiano de coordenadas inteiras.
Três quaisquer desses pontos não estão alinhados, logo formam dez segmentos.
Mostre que pelo menos um dos pontos de intersecção desses segmentos é um
ponto, também, de coordenadas inteiras.
Desde já agradeço.

-- 

Pedro Jerônimo S. de O. Júnior

Professor de Matemática

Geo João Pessoa – PB


[obm-l] Re: [obm-l] Re: [obm-l] Essa ainda não consegui!!!

2011-07-24 Por tôpico Pedro Júnior
Exatamente caríssimo Ralph, tens razão, é que estava tentanto lembrar do
problema e fui escrevendo, mas vc me fez lembrar direitinho, como sempre!!!
Parabéns.

Em 24 de julho de 2011 11:23, Ralph Teixeira ralp...@gmail.com escreveu:

 Ah... aposto que o problema original era para mostrar que um dos PONTOS
 MEDIOS desses 10 segmentos tem coordenadas inteiras, nao? Ai tudo faz
 sentido: basta olhar a paridade de ambas as coordenadas. Ha 4 classes de
 possibilidades: (Par,Par), (Par, Impar), (Impar, Par), (Impar, Impar). Como
 voce tem 5 pontos, pombas, tem que haver dois deles dentro da mesma
 classe, digamos, X e Y. Mas entao as coordenadas de X+Y serao ambas pares,
 isto eh, as coordenadas do ponto medio (X+Y)/2 serao inteiras.

 Aposto 10 pratas que era esse o problema! Em dolar! :)

 Abraco,
 Ralph

 2011/7/24 Pedro Júnior pedromatematic...@gmail.com

 Sejam A, B, C, D e E pontos do plano cartesiano de coordenadas inteiras.
 Três quaisquer desses pontos não estão alinhados, logo formam dez segmentos.
 Mostre que pelo menos um dos pontos de intersecção desses segmentos é um
 ponto, também, de coordenadas inteiras.
 Desde já agradeço.

 --

 Pedro Jerônimo S. de O. Júnior

 Professor de Matemática

 Geo João Pessoa – PB





-- 

Pedro Jerônimo S. de O. Júnior

Professor de Matemática

Geo João Pessoa – PB


Re: [obm-l] A procura de um livro! (off-topic)

2011-07-21 Por tôpico Pedro Júnior
Olha, muitíssimo obrigado, o arquivo será sim útil!


Em 20 de julho de 2011 09:59, Carlos Nehab ne...@infolink.com.br escreveu:

 **
 Tem na Amazon, João,

 Abraços,
 Nehab

 Em 20/7/2011 08:13, Pedro Júnior escreveu:

 Alguém poderia me indicar algum site que tenha o livro:
 L. E. Dickson, Algebras and their Arithmetics, University of Chicago Press,
 1923
 p.s.: poderia ser para download, pois pela data acho que não tem mais para
 vender!
 --

 Pedro Jerônimo S. de O. Júnior

 Professor de Matemática

 Geo João Pessoa – PB





-- 

Pedro Jerônimo S. de O. Júnior

Professor de Matemática

Geo João Pessoa – PB


[obm-l] A procura de um livro! (off-topic)

2011-07-20 Por tôpico Pedro Júnior
Alguém poderia me indicar algum site que tenha o livro:
L. E. Dickson, Algebras and their Arithmetics, University of Chicago Press,
1923
p.s.: poderia ser para download, pois pela data acho que não tem mais para
vender!
-- 

Pedro Jerônimo S. de O. Júnior

Professor de Matemática

Geo João Pessoa – PB


[obm-l] Re: [obm-l] Re: [obm-l] Equação de variáveis inteiras

2011-06-02 Por tôpico Pedro Júnior
Exatamente Ralph, quando utilizei as leis de seno e cosseno achei o cosseno
de um dos ânulos maior que 1 o que torna inválido o problema.
Falou Ralph, comunicarei ao comitê olímpico da UFCG.
Abraços.

Em 30 de maio de 2011 14:06, Ralph Teixeira ralp...@gmail.com escreveu:

 2) Com este enunciado, não há triângulo nestas condições...

 Tracei a bissetriz interna AX do ângulo A, fiz CX=x. Note que AXC é
 isósceles, então AC=2xcosC, então BC=xcosCx=XC. Em outras palavras, a
 bissetriz interna AX corta o lado BC *fora* de BC, absurdo.

 Não seria ângulo C=2A? Aí seria um triângulo 30-60-90 bonitinho...

 3) Bom, se x=0 então y=7, e vice-versa. Se x=1, note que y não dá inteiro,
 e vice-versa. Vamos então supor logo que x,y=2 no resto do problema.

 Eu passei o x^2 pro lado de lá para fatorar:
 (xy-7+x)(xy-7-x)=y^2.

 (Agora, minha intuição me diz que, em geral, ambos xy-7+x e xy-7-x devem
 ser BEM maiores que y, então isto vai restringir o problema... AH-HA!)

 Note que xy-7-xxy-7+x (pois x=2). Assim, devemos ter xy-7-xy (caso
 contrário, ambos os fatores seriam maiores ou iguais a y, e então o produto
 seria maior que y^2).

 xy-7+x-y0
 (x-1)(y+1)=5

 Como x-1=1, devemos ter y+1=5, isto é, basta analisar y=2,3,4.

 Abraço,
 Ralph
 2011/5/30 Pedro Júnior pedromatematic...@gmail.com

 Questões 02 e 03 da 2ª Parte da XXIV OCM - 2011 Nível 03, que ocorreu
 neste último sábado dia 28 de Maio:

 *02.* Um triângulo ABC é tal que o ângulo A=2C e AC = 2BC.. Mostre que
 este triângulo é retângulo.
 Usei a lei dos senos e lei dos cossenos mas não consegui concluir, favor
 quem tiver alguma ideia, contribuir...

 *03.* Determine todos os pares de inteiros não negativos que são soluções
 da equação (xy - 7)^2 = x^2 + y^2.

 Sem nenhuma estratégia descobrir que os pares (3,4); (4,3); (0,7); (7,0)
 satisfazem tal equação.
 Tentei enxergar o teo. de Pitágoras, fazendo x e y como catetos e xy - 7
 como hipotenusa.
 Há alguma resolução algébrica, alguma substituição que torne a equação com
 uma só incórnita?

 Desde já aradeço.

 --

 Pedro Jerônimo S. de O. Júnior

 Professor de Matemática

 Geo João Pessoa – PB





-- 

Pedro Jerônimo S. de O. Júnior

Professor de Matemática

Geo João Pessoa – PB


[obm-l] Equação de variáveis inteiras

2011-05-30 Por tôpico Pedro Júnior
Questões 02 e 03 da 2ª Parte da XXIV OCM - 2011 Nível 03, que ocorreu neste
último sábado dia 28 de Maio:

*02.* Um triângulo ABC é tal que o ângulo A=2C e AC = 2BC.. Mostre que este
triângulo é retângulo.
Usei a lei dos senos e lei dos cossenos mas não consegui concluir, favor
quem tiver alguma ideia, contribuir...

*03.* Determine todos os pares de inteiros não negativos que são soluções da
equação (xy - 7)^2 = x^2 + y^2.

Sem nenhuma estratégia descobrir que os pares (3,4); (4,3); (0,7); (7,0)
satisfazem tal equação.
Tentei enxergar o teo. de Pitágoras, fazendo x e y como catetos e xy - 7
como hipotenusa.
Há alguma resolução algébrica, alguma substituição que torne a equação com
uma só incórnita?

Desde já aradeço.

-- 

Pedro Jerônimo S. de O. Júnior

Professor de Matemática

Geo João Pessoa – PB


[obm-l] Números Inteiros

2011-05-29 Por tôpico Pedro Júnior
10ª Questão da Olimpíada Campinense de Matemática - 2011 - Realizada em 28
de Maio de 2011.

10. Qual da quantidade de números inteiros positivos de dois algarismos tais
que a diferença entre o número e o produto seja 12.

-- 

Pedro Jerônimo S. de O. Júnior

Professor de Matemática

Geo João Pessoa – PB


[obm-l] Re: [obm-l] Re: [obm-l] Re: [obm-l] RE: [obm-l] Número de partições de um conjunto

2011-05-22 Por tôpico Pedro Júnior
Falou cara muitíssimo obriado.
Olá Paulo Santa Rita, há quanto tempo não conversávamos não é mesmo?
Olha meu erro foi fazer o r variar de 1 até n-r salvo o engano, depois
somei todos os resultados, por isso deu aquele somatório. Mas sua solução
como sempre foi brilhante.
Abração e muito obrigado.

Em 20 de maio de 2011 12:54, Alessandro Madruga Correia 
amcorr...@viaconnect.com.br escreveu:

 Olá,

 me intrometendo...


 Caro Wily como fizestes para aparecer a imagem?
 Paulo volto a falar contigo!


 Ele utilizou esse site, http://www.codecogs.com/latex/htmlequations.php


 --
   ,= ,-_-. =.   [o] Alessandro Madruga Correia
  ((_/)o o(\_))  Viaconnect -- Suporte Técnico +55 (54) 4009 3444
   `-'(. .)`-'   Certamente, tenho arriscado minha saúde algumas vezes pelo
   \_/   excesso de trabalho, mas e daí? Somente os repolhos não têm
 nervos, nem preocupações. E o que conseguem com seu
 bem-estar
 perfeito? (Carl Gustav Jacob Jacobi)




-- 

Pedro Jerônimo S. de O. Júnior

Professor de Matemática

Geo João Pessoa – PB


[obm-l] Re: [obm-l] Re: [obm-l] RE: [obm-l] Número de partições de um conjunto

2011-05-20 Por tôpico Pedro Júnior
Caro Wily como fizestes para aparecer a imagem?
Paulo volto a falar contigo!

Em 19 de maio de 2011 15:45, Willy George Amaral Petrenko 
wgapetre...@gmail.com escreveu:

 Acho que faz sentido ao invés de usar LaTex, usar a imagem, assim fica mais
 acessível:
 Acho que todo mundo vai conseguir ler (corrijam-me se eu estiver errado).

 Bem, me parece que vc quis resolver o problema, não para r e s, mas para
 quaisquer 2 conjuntos. A resposta do Paulo está correta para o que pede o
 enunciado. Se você quiser calcular para quaisquer 2 conjuntos, tem que tomar
 cuidado com o possível termo
  pois ele não está sendo contado 2 vezes para vc fazer 1/2*.




-- 

Pedro Jerônimo S. de O. Júnior

Professor de Matemática

Geo João Pessoa – PB


[obm-l] Número de partições de um conjunto

2011-05-19 Por tôpico Pedro Júnior
No primeiro problema cheguei a algo do tipo 1/2\cdot [ C_{n}^{1} \cdot
(2^{n-1}-1) + C_{n}^{2} \cdot (2^{n-2}-1) + C_{n}^{3} \cdot (2^{n-3}-1)
+...+C_{n}^{n-1} ]
queria saber se alguém sabe opinaar se estou no caminho correto.
Abraços.

1. Seja X um conjunto com n elementos. Calcule o número de escolhas
possíveis
de dois subconjuntos disjuntos de r e s elementos, respectivamente. [E
se r = s?]


2. O mesmo exercício anterior mas em que os dois subconjuntos possam
intersectar-se
num único elemento.

-- 

Pedro Jerônimo S. de O. Júnior

Professor de Matemática

Geo João Pessoa – PB


[obm-l] Re: [obm-l] Re: [obm-l] Função Bijetiva

2011-05-13 Por tôpico Pedro Júnior
Olá a todos, seguinte o livro que foi retirado o problema é Set Theory, cujo
autor Charles C. Pinter, Bucknell Unniversity, publicado pela Addison-Wesley
Publishing Company na década de 70.

Problema:

A~B iff A is one-to-one correspondence with B.

1. Suppose that A ~ B, a \in A, and b \in B. Prove that (A - {a}) ~ (B -
{b}).

2. Suppose that A ~ B, C ~ D, C \cup A and D \cup B. Prove that (A - C) ~ (B
- D).

De fato, havia esquecido da bijeção entre C e D.





Em 9 de maio de 2011 23:23, Ralph Teixeira ralp...@gmail.com escreveu:

 É, tome A=B=D=Z e C=N.

 Então existe uma bijeção I:A-B (a identidade);
 e existe uma bijeção f:C-D (levando {0,1,2,...} em
 {0,-1,1,-2,2,-3,...}, respectivamente)

 Mas não há bijeção de A-C=Z_{-} em B-D=vazio!

 Abraço,
 Ralph

 =
 Instruções para entrar na lista, sair da lista e usar a lista em
 http://www.mat.puc-rio.br/~obmlistas/obm-l.html
 =




-- 

Pedro Jerônimo S. de O. Júnior

Professor de Matemática

Geo João Pessoa – PB


[obm-l] Re: [obm-l] Re: [obm-l] Re: [obm-l] Re: [obm-l] Função Bijetiva

2011-05-13 Por tôpico Pedro Júnior
Em 13 de maio de 2011 13:42, Bernardo Freitas Paulo da Costa 
bernardo...@gmail.com escreveu:

 2011/5/13 Pedro Júnior pedromatematic...@gmail.com:
  Olá a todos, seguinte o livro que foi retirado o problema é Set Theory,
 cujo
  autor Charles C. Pinter, Bucknell Unniversity, publicado pela
 Addison-Wesley
  Publishing Company na década de 70.
 
  Problema:
 
  A~B iff A is one-to-one correspondence with B.
 
  1. Suppose that A ~ B, a \in A, and b \in B. Prove that (A - {a}) ~ (B -
  {b}).
 
  2. Suppose that A ~ B, C ~ D, C \cup A and D \cup B. Prove that (A - C) ~
 (B
  - D).
 
  De fato, havia esquecido da bijeção entre C e D.
 Como eu disse e o Ralph provou, ainda falta alguma coisa. Tipo uma
 hipótese de que C e D são finitos, para você poder usar recorrência da
 propriedade 1. ; sem isso, continua falso.
 --
 Bernardo Freitas Paulo da Costa
 Não Bernardo, veja que entre C e D existe uma bijeção, ou seja, esta é a
 hipótese que faltava, agora falta provar!


   Em 9 de maio de 2011 23:23, Ralph Teixeira ralp...@gmail.com
 escreveu:
 
  É, tome A=B=D=Z e C=N.
 
  Então existe uma bijeção I:A-B (a identidade);
  e existe uma bijeção f:C-D (levando {0,1,2,...} em
  {0,-1,1,-2,2,-3,...}, respectivamente)
 
  Mas não há bijeção de A-C=Z_{-} em B-D=vazio!
 
  Abraço,
 Ralph
 
 
  --

 =
 Instruções para entrar na lista, sair da lista e usar a lista em
 http://www.mat.puc-rio.br/~obmlistas/obm-l.html
 =




-- 

Pedro Jerônimo S. de O. Júnior

Professor de Matemática

Geo João Pessoa – PB


[obm-l] Nº de funções sobrejetoras

2011-03-28 Por tôpico Pedro Júnior
Um colega me propôs o seguinte problema, e não consegui modelar:
Seja A um conjunto com n elementos e seja B um conjunto com m elementos, com
n = m. Quantas funções sobrejetoras, f : A -- B, podemos formar?

-- 

Pedro Jerônimo S. de O. Júnior

Professor de Matemática

Geo João Pessoa – PB


Re: [obm-l] Trigonometria

2010-11-16 Por tôpico Pedro Júnior
Olá Carlos você está correto!!!
par que o problema ficasse correto bastava escrever 2cos20º - 1/*2*cos80º
note que faltou esse dois muitiplicando o cos80º. Problema que de fato sua
resolução passa pela cúbica citada no em seu texto. Porém, muitíssimo
obrigado pela participação.


Agora, será que você ou alguém poderia me ajudar noutro problema da mesma
prova:

4. Seja n um inteiro maior que 2. Se c é a hipotenusa de um triângulo
retângulo e a e b são seus catetos, prove que c^n  a^n + b^n.

Desde já agradeço.
Pedro Jr
João Pessoa - PB
Abraços.


Em 15 de novembro de 2010 08:42, Carlos Nehab ne...@infolink.com.brescreveu:

  Oi, Pedro,

 Infelizmente o enunciado está errado.
 Mas para você não ficar triste, tente resolver algo parecido e correto:

 2cos 20 - 1/ (2cos 40 -1)  é um inteiro...

 Abraços
 Carlos Nehab

 Dica: este negócio de 20, 40 e 80 graus muitas vezes acabam em samba se
 você usar as expressões de arco triplo, pois linhas trigonométricas desse
 arcos se expressam em termos de raízes de uma equação cúbica..., que no
 fundo é o que as expressões do arco triplo nos mostram...  Se você conhecer
 Cardano, poderá inclusive se divertir (?) explicitando os senos e cossenos
 destes arcos.
 Vários problemas interessantes já circularam por aqui com estes malditos
 e instigantes ângulos...  Além disso um estudo do eneágono e do octadecágono
 (18 lados) também será fascinante para quem gosta destes angulozinhos
 decididamente desafiadores.


 Em 15/11/2010 00:00, Pedro Júnior escreveu:

 Parece simples mais ainda não consegui exergar o caminho.
 Usei tansformações, forma exponencial dos complexos, combinei várias
 transformações, etc, só ainda não dei um tratamento geométrico..


 Vejam: Mostre que 2cos20º - 1/cos80º é um inteiro.


 Abraços.

 Pedro Júnior
 João Pessoa - PB





Re: [obm-l] Trigonometria

2010-11-16 Por tôpico Pedro Júnior
Isso Arlane muitíssimo obrigado...

Em 16 de novembro de 2010 08:33, Arlane Manoel S Silva ar...@usp.brescreveu:

   Um Pedro, uma prova desse resultado pode ser feita por indução em n2.
 Como c
  é hipotenusa temos ca e cb. Para n=3 temos
   c^3=c(a^2+b^2)=c.a^2+c.b^2a.a^2+b.b^2=a^3+b^3.

   Acho que vc pode continuar a prova.

   A.
 Citando Pedro Júnior pedromatematic...@gmail.com:

  Olá Carlos você está correto!!!
 par que o problema ficasse correto bastava escrever 2cos20º - 1/*2*cos80º

 note que faltou esse dois muitiplicando o cos80º. Problema que de fato
 sua
 resolução passa pela cúbica citada no em seu texto. Porém, muitíssimo
 obrigado pela participação.


 Agora, será que você ou alguém poderia me ajudar noutro problema da mesma
 prova:

 4. Seja n um inteiro maior que 2. Se c é a hipotenusa de um triângulo
 retângulo e a e b são seus catetos, prove que c^n  a^n + b^n.

 Desde já agradeço.
 Pedro Jr
 João Pessoa - PB
 Abraços.


 Em 15 de novembro de 2010 08:42, Carlos Nehab ne...@infolink.com.br
 escreveu:

  Oi, Pedro,

 Infelizmente o enunciado está errado.
 Mas para você não ficar triste, tente resolver algo parecido e correto:

 2cos 20 - 1/ (2cos 40 -1)  é um inteiro...

 Abraços
 Carlos Nehab

 Dica: este negócio de 20, 40 e 80 graus muitas vezes acabam em samba se
 você usar as expressões de arco triplo, pois linhas trigonométricas desse
 arcos se expressam em termos de raízes de uma equação cúbica..., que no
 fundo é o que as expressões do arco triplo nos mostram...  Se você
 conhecer
 Cardano, poderá inclusive se divertir (?) explicitando os senos e
 cossenos
 destes arcos.
 Vários problemas interessantes já circularam por aqui com estes
 malditos
 e instigantes ângulos...  Além disso um estudo do eneágono e do
 octadecágono
 (18 lados) também será fascinante para quem gosta destes angulozinhos
 decididamente desafiadores.


 Em 15/11/2010 00:00, Pedro Júnior escreveu:

 Parece simples mais ainda não consegui exergar o caminho.
 Usei tansformações, forma exponencial dos complexos, combinei várias
 transformações, etc, só ainda não dei um tratamento geométrico..


 Vejam: Mostre que 2cos20º - 1/cos80º é um inteiro.


 Abraços.

 Pedro Júnior
 João Pessoa - PB







 --
Arlane Manoel S Silva
  Departamento de Matemática Aplicada
 Instituto de Matemática e Estatística-USP

 =
 Instruções para entrar na lista, sair da lista e usar a lista em
 http://www.mat.puc-rio.br/~obmlistas/obm-l.html
 =



Re: [obm-l] Trigonometria

2010-11-15 Por tôpico Pedro Júnior
Olha esse problema foi da Olimpíada Pessoense de Matemática 2010 (João
Pessoa - PB), de fato não fiz as contas usando uma máquina, porém a dúvida
é, será que a máquina não fez arredondamentos que não torne a diferença um
número inteiro?
De fato cheguei a desconfiar que tal problema apresenta falhas em sua
edição.
Nesse momento, diante das colocações feitas, como mostro que não é inteiro?

Em 15 de novembro de 2010 01:19, Ivan lopes lopesivan@gmail.comescreveu:

 2cos20º - 1/cos80º = -3,879385242
 nops!

 2010/11/15 Marcos Valle marcos.vall...@gmail.com

 2cos20º - 1/cos80º
 2cos20° - 1/sen10°
 2(1 - 2sen²10°) - 1/sen10°
 (2sen10° - 4sen³10° - 1)/sen10°
 (sen30° - sen10° - 1)/sen10°
 (-1/2 - sen10°)/sen10°
 -1 - 1/(2sen10°)

 Pode usar o que quiser, vai ser difícil de achar um inteiro aí =]



 Em 15 de novembro de 2010 00:00, Pedro Júnior 
 pedromatematic...@gmail.com escreveu:

 Parece simples mais ainda não consegui exergar o caminho.
 Usei tansformações, forma exponencial dos complexos, combinei várias
 transformações, etc, só ainda não dei um tratamento geométrico..


 Vejam: Mostre que 2cos20º - 1/cos80º é um inteiro.




 Abraços.


 Pedro Júnior
 João Pessoa - PB




 --
 Marcos Valle
 Instituto Militar de Engenharia - IME
 1° ano A - básico





[obm-l] Trigonometria

2010-11-14 Por tôpico Pedro Júnior
Parece simples mais ainda não consegui exergar o caminho.
Usei tansformações, forma exponencial dos complexos, combinei várias
transformações, etc, só ainda não dei um tratamento geométrico..


Vejam: Mostre que 2cos20º - 1/cos80º é um inteiro.




Abraços.


Pedro Júnior
João Pessoa - PB


[obm-l] Re: [obm-l] Re: [obm-l] Re: [obm-l] RES: [obm-l] Re: [obm-l]RES: [obm-l] Re: [obm-l] Re: [obm-l] 125 perguntas a inda não respondidas pela Ciência

2010-11-10 Por tôpico Pedro Júnior
CERTEZA UMA DELAS É DA OBM!!!
MINHA NOSSA!!!

Em 10 de novembro de 2010 19:21, Willy George do Amaral Petrenko 
wgapetre...@gmail.com escreveu:

 Bem, parece que eu disse besteira mesmo quanto a gravidade.


 Ao contrário do que possa ter parecido eu não disse que: não cabe a
 investigação filosófica na ciência. A ciência e a filosofia estão ligadas,
 uma pode motivar a outra, mas ainda assim elas são diferentes (caso
 contrário não existiriam 2 nomes).

 A fronteira pode até ser obscura, mas coisas como porque existe algo? ou
 porque existe padrão na natureza? certamente não fazem parte da ciência.

 Ainda vou dar uma olhada ainda no link...

 Abc



Re: [obm-l] Provando Continuidade

2010-06-21 Por tôpico Pedro Júnior
Vê o livro do Elon vol. 02, lá tem uma série de contra-exemplos como estes
que o Ralph falou, vê também o APOSTOL talvez o melhor do assunto.
Abraços.

Em 20 de junho de 2010 13:03, Gustavo Souza 
gustavoandre2006s...@yahoo.com.br escreveu:

 Nossa, vendo vocês comentarem isso vejo que estou muito fraquinho nessa
 matéria, estou estudando desde o começo do ano pelo livro do Swokowski -
 Calculo com Geometria Analítica, Vol. II e mesmo assim não vejo grandes (nem
 bons) resultados.

 Será que alguem teria algum material pra me recomendar (livros, sites,
 apostilas, qlqr coisa)?

 Abraços e Obrigado

 --
 *De:* Ralph Teixeira ralp...@gmail.com
 *Para:* obm-l@mat.puc-rio.br
 *Enviadas:* Domingo, 20 de Junho de 2010 11:44:03
 *Assunto:* Re: [obm-l] Provando Continuidade

 Eh, do jeito que foi feito, estah errado duas vezes.

 Primeiro, porque nem todas as retas do plano sao da forma y=tx -- tem a
 reta x=0, que nao pode ser escrita assim.

 Segundo, porque ha exemplos classicos de funcoes que sao continuas por
 todas as retas mas nao sao continuas. O que eu tenho no meu bolso eh este:

 f(x,y) = 2x^2.y/(x^4+y^2) para (x,y)(0,0)
 f(0,0)=0

 Eh facil ver que:

 i) Se x=0 ou y=0, f eh identicamente nula.
 ii) Se y=ax com a0, f(t,at)=2at^3/(t^4+a^2.t^2)=2at/(t^2+a^2) - 0 quando
 t-0.
 iii) Porem, f(t,t^2)=1, entao quanto t-0, (t,t^2) se aproxima da origem e
 f(t,t^2)-1.

 Eh muito bom desenhar as curvas de nivel deste exemplo, o grafico se
 possivel, ver como as retas cortam as curvas de nivel, e entender o que
 estah acontecendo.

 Abraco, Ralph.
 2010/6/20 Bruno França dos Reis bfr...@gmail.com

 Do jeito que está na foto, essa resposta está errada.

 A afirmação (II) está errada. É errado concluir que o limite de f(x, y)
 para (x, y) tendendo a (0, 0) é 0, usando apenas o que foi afirmado antes.

 O que vc provou é que o limite de f(x, y) para (x, y) tendendo a (0, 0) 
 *através
 de qualquer reta que passa pela origem* vale 0. Mas vc precisa provar que
 f(x, y) tende a (0, 0) quando (x, y) tende a (0, 0) por *todo e qualquer
 caminho existente*.


 Um jeito simples de resolver este exercício é tentar fatorar (x^3 + y^3).
 Dica: (x^3 + y^3) = (x + y)*(...)



 --
 Bruno FRANÇA DOS REIS

 msn: brunoreis...@hotmail.com
 skype: brunoreis666
 tel: +55 11 9961-7732

 http://brunoreis.com
 http://brunoreis.com/tech (en)
 http://brunoreis.com/blog (pt)

 GPG Key: http://brunoreis.com/bruno-public.key

 e^(pi*i)+1=0


 2010/6/19 Gustavo Souza gustavoandre2006s...@yahoo.com.br

   Ola a todos, sei que esse exercício foge um pouco dos demais aqui da
 lista, mas se alguem puder me dar alguma opinião. Desde ja agradeço.

 *Enunciado :* Verifique a continuidade (a função esta presente no link
 abaixo)

 http://img257.imageshack.us/img257/807/pergunta.png

 *Possível Resposta *: (No link abaixo)

 http://img199.imageshack.us/img199/8342/resposta.png


 A minha duvida é: Essa resposta ,do jeito que esta na foto, é considerada
 correta? Se não pq? Se sim pq também?

 Obrigado e Abraços










Re: [obm-l] Essa vale a pena!

2010-04-11 Por tôpico Pedro Júnior
primeiramente, separe a soma em duas pela associatividade, (1+sqrt cub(2)) +
(sqrt cub(4)) Agora use a identidade
a^3 + b^3 = (a+b)(a^2 - ab + b^2), tal soma em baixo é o fator (a + b), e
depois repete o procedimento.
Abraços

Em 10 de abril de 2010 23:09, adriano emidio
adrianoemi...@yahoo.com.brescreveu:

 Lembro quando estava na faculdade de que meu professor de Álgebra
 racionalizou:

 1/(1+3 raizcubica de 2+raiz cubica de 4)

 só que perdi as notas de aula e não consigo resolver mais. Alguém pode
 tentar? Valeu e abraços a todos!




Re: [obm-l] numero irracional

2010-03-23 Por tôpico Pedro Júnior
Veja livro do Elon: Meu Professor de Matemática
Pedro Júnior

Em 23 de março de 2010 14:11, Adalberto Dornelles
aadornell...@gmail.comescreveu:

 É racional.
 0,... = 1

 Adalberto


 Em 23 de março de 2010 13:45, Olinto Araújo olinto...@gmail.com
 escreveu:
  O número 0, é irracional ou racional ?
 
  Agradeço
 
  Olinto
 

 =
 Instruções para entrar na lista, sair da lista e usar a lista em
 http://www.mat.puc-rio.br/~obmlistas/obm-l.htmlhttp://www.mat.puc-rio.br/%7Eobmlistas/obm-l.html
 =



[obm-l] Fwd: Repunit

2010-02-18 Por tôpico Pedro Júnior
-- Mensagem encaminhada --
De: Pedro Júnior pedromatematic...@gmail.com
Data: 15 de fevereiro de 2010 17:01
Assunto: Repunit
Para: obm-l obm-l@mat.puc-rio.br


Prove que: *111...1 (com n dígitos iguais a 1) é divisível por 41 se, e
somente se n é divisível por 5*.


Desde já agradeço!!!

Abraços.

Pedro Jr


[obm-l] Repunit

2010-02-18 Por tôpico Pedro Júnior
Prove que: *111...1 (com n dígitos iguais a 1) é divisível por 41 se, e
somente se n é divisível por 5*.


Desde já agradeço!!!

Abraços.

Pedro Jr


Re: [obm-l] Fwd: Repunit

2010-02-18 Por tôpico Pedro Júnior
Exatamente isso que estava a procura...
Vou me inteirar, mas obrigadíssimo!!!

Abraços

Pedro Jr

João Pessoa - PB

Em 18 de fevereiro de 2010 20:57, Tiago hit0...@gmail.com escreveu:

 Muito boa sua solução!

 2010/2/18 Cesar Kawakami cesarkawak...@gmail.com

 Uma maneira alternativa é mostrar que n múltiplo de 5 se, e somente
 se, 41 divide (10^n - 1) / 9, que é equivalente a 41 divide 10^n - 1
 pois 9 e 41 são primos entre si. Como a ordem de 10 módulo 41 é 5
 (10^5 deixa resto 1 módulo 41 e nenhuma das potências anteriores o
 faz, e isso é diretamente checável), temos que 41 divide 10^n - 1 se,
 e somente se, n é divisível por 5.



 []'s
 Cesar

 2010/2/18 Albert Bouskela bousk...@msn.com:
  Novamente, olá!
 
 
 
  Abaixo, fiz a complementação para k=4. Para k=1, 2, 3, é só seguir a
 mesma
  metodologia
 
 
 
  Albert Bouskela
 
  bousk...@msn.com
 
 
 
  De: owner-ob...@mat.puc-rio.br [mailto:owner-ob...@mat.puc-rio.br] Em
 nome
  de Albert Bouskela
  Enviada em: quinta-feira, 18 de fevereiro de 2010 12:34
  Para: obm-l@mat.puc-rio.br
  Assunto: RES: [obm-l] Fwd: Repunit
 
 
 
  Olá!
 
 
 
  Por Indução Finita, é fácil verificar que:
 
 
 
  Se “n” é múltiplo de 5, então 111...111 (com “n” dígitos iguais a 1) é
  múltiplo de 41.
 
 
 
  Lá vai:
 
 
 
  1.   Verifica-se que 1 é múltiplo de 41 (271*41=1).
 
  2.   Hipótese de Indução: [111...111 (com “n” dígitos iguais a 1 e “n”
  múltiplo de 5)] é múltiplo de 41.
 
  3.   Então, mostra-se que (próximo passo da demonstração por Indução
  Finita):
 
  {[111...111 (com “n” dígitos iguais a 1 e “n” múltiplo de 5)]*10^5 +
 1}
  é múltiplo de 41.
 
  Fácil: [111...111 (com “n” dígitos iguais a 1, “n” múltiplo de 5)]*10^5
 é
  múltiplo de 41 (consequência imediata da própria Hipótese de Indução); e
  1 é múltiplo de 5 (ver passo 1).
 
 
 
  Falta verificar que:
 
  Se “n” é igual a (5m + k, k=1, 2, 3, 4), então 111...111 (com “n”
 dígitos
  iguais a 1) NÃO é múltiplo de 41.
 
  Dá trabalho (são 4 verificações), mas parece-me que seja igualmente
 fácil...
 
 
 
  Para k=4:
 
  n = 5m + 4
 
  a.   [111...111 (com “5m” dígitos iguais a 1)] é múltiplo de 41. Já foi
  verificado acima, já que “5m” é – obviamente – múltiplo de 5.
 
  b.   [111...111 (com “5m” dígitos iguais a 1)] = 41p (um múltiplo de 41)
 
  c.   [111...111 (com “5m” dígitos iguais a 1)]*10^4 + 
 
  d.   [111...111 (com “5m” dígitos iguais a 1)]*10^4 é múltiplo de 41
 (ver
  passo c);  = 41*27 + 4
 
  e.   [111...111 (com “5m” dígitos iguais a 1)]*10^4 +  = 41(p+27) +
 4
 
  f.   Logo, [111...111 (com “5m” dígitos iguais a 1)]*10^4 +  tem
 resto 4
  na divisão por 41. Logo, [111...111 (com “5m” dígitos iguais a 1)]*10^4
 +
   NÃO é múltiplo de 41.
 
 
 
  Agora, é só fazer para k=1, 2, 3.
 
 
 
  Albert Bouskela
 
  bousk...@msn.com
 
 
 
  De: owner-ob...@mat.puc-rio.br [mailto:owner-ob...@mat.puc-rio.br] Em
 nome
  de Pedro Júnior
  Enviada em: quarta-feira, 17 de fevereiro de 2010 10:07
  Para: obm-l
  Assunto: [obm-l] Fwd: Repunit
 
 
 
 
 
  -- Mensagem encaminhada --
  De: Pedro Júnior pedromatematic...@gmail.com
  Data: 15 de fevereiro de 2010 17:01
  Assunto: Repunit
  Para: obm-l obm-l@mat.puc-rio.br
 
 
  Prove que: 111...1 (com n dígitos iguais a 1) é divisível por 41 se, e
  somente se n é divisível por 5.
 
 
  Desde já agradeço!!!
 
  Abraços.
 
  Pedro Jr
 
 

 =
 Instruções para entrar na lista, sair da lista e usar a lista em
 http://www.mat.puc-rio.br/~obmlistas/obm-l.htmlhttp://www.mat.puc-rio.br/%7Eobmlistas/obm-l.html
 =




 --
 Tiago J. Fonseca
 http://legauss.blogspot.com



[obm-l] Re: [obm-l] Problema de máximo!!!

2009-11-04 Por tôpico Pedro Júnior
Obrigado e aproveito a oportunidade para te parabenizar, pelas brilhantes
intervenções que tem feito ao longo desses anos, contribuindo assim para uma
discussão de qualidade voltada pro aprendizado, sem perder qualidade
didática.
Parabéns Professor


2009/11/4 Carlos Nehab ne...@infolink.com.br

 Oi, Pedro,

 Da igualdade (b + c)^2 = b^2 + c^2 + 2bc
 obtemos (b + c)^2 = a^2 + 2bc, onde a é a hipotenusa,
 Dai b+c é máximo quando bc for máximo e ai a solução é imediata, pois
 bc = ah (a fixo) e então bc é máximo quando a altura for máxima.

 Abraços,
 Nehab

 Pedro Júnior escreveu:

  Prove que, entre todos os triângulos retângulos de catetos a e b e
 hipotenusa c fixada, o que tem maior soma dos catetos
 S = a + b é o triângulo isósceles.


 =
 Instruções para entrar na lista, sair da lista e usar a lista em
 http://www.mat.puc-rio.br/~obmlistas/obm-l.htmlhttp://www.mat.puc-rio.br/%7Eobmlistas/obm-l.html
 =



[obm-l] Re: [obm-l] Re: [obm-l] Re: [obm-l] RE: [obm-l] Prob lema de máximo!!!

2009-11-03 Por tôpico Pedro Júnior
Rapaz, que discussão sadia e legal, extremamente didática ao mesmo tempo em
que há um tom de pesquisa. Armas são levantadas, de maneira que surja a
descoberta!
Olha pessoal, essas últimas discussões estão exatamente às voltas de onde
parei, daí decidi postar na lista. Maximizar a soma de lados, dado que a
soma dos quadrados desses lados é constante,  nunca pensei que fosse tão
complicado (me refiro ao nível de discussão desta lista) sem o uso de
trigonometria (pois são alunos do 9º ano).

2009/11/3 luiz silva luizfelipec...@yahoo.com.br

 Tem a relação de áreas abaixo :

 S1 = abc/4R ; S2 = x^2c/4R

 x^2c/4R  abc/4R

 x^2  ab

 mas ainda não vejo como usá-laalém disso,  de pitágoras, podemos, tb,
  tirar o resultado :

 2x^2= c^2

 a^2+b^2=c^2

 x^2 = (a^2+b^2)/2

 x = [(a^2+b^2)/2]^(1/2)
 2x= 2 [(a^2+b^2)/2]^(1/2)  a+b

 2a^2+2b^2a^2+2ab+b^2
 a^2-2ab+b20
 (a-b)^20

 Isto é sempre verdade, exceto para a=b.

 Abs
 Felipe
 --- Em *ter, 3/11/09, Bernardo Freitas Paulo da Costa 
 bernardo...@gmail.com* escreveu:


 De: Bernardo Freitas Paulo da Costa bernardo...@gmail.com
 Assunto: [obm-l] Re: [obm-l] RE: [obm-l] Problema de máximo!!!
 Para: obm-l@mat.puc-rio.br
 Data: Terça-feira, 3 de Novembro de 2009, 13:03

 2009/11/3 luiz silva 
 luizfelipec...@yahoo.com.brhttp://br.mc657.mail.yahoo.com/mc/compose?to=luizfelipec...@yahoo.com.br
 
  Ola Pessoal,
 Oi Luiz e outros !

  Não sei se o meu argumento é válido, mas analisando a questão reparamos
 que temos um segmento de reta fixo.
 exatamente !

  Se considerarmos os triângulos formados pelas envoltórias (que são os
 lados diferentes) e o segmento fixo veremos que o triângulo de maior altura
 é o triângulo isósceles. Isto quer dizer que é o triângulo com maior área.
 Muito bem ! Ah, tem uma coisa a mais, os ápices dos triângulos formam
 uma circunferência !

  Como a base é a mesma (hipotenusa), para envolver uma maior área são
 necessários dois segmentos que, somados, serão maiores que a soma dos
 segmentos que envolvem uma área menor, dada a mesma base. Assim, como a base
 é igual em todos os triãngulos, o de maior perímetro será aquele com a maior
 soma dos outros dois lados, ou seja, o triangulo retangulo isósceles.

 Essa parte da intuição é ótima, mas eu acho que precisa formalizar.
 Quando eu mandei a minha idéia da construção geométrica, era para
 tentar ver alguma coisa além da pura trigonometria, e usar algo como
 MA = MG para a+b e a*b de alguma forma esperta. A conexão que você
 deu para a área permite usar p(p-a)(p-b)(p-c), e eu acho que quando
 a+b for máximo, c fixo, deve dar pra provar que a=b.

  Abs
  Felipe

 Quem continua ?
 --
 Bernardo Freitas Paulo da Costa

 =
 Instruções para entrar na lista, sair da lista e usar a lista em
 http://www.mat.puc-rio.br/~obmlistas/obm-l.htmlhttp://www.mat.puc-rio.br/%7Eobmlistas/obm-l.html
 =


 --
 Veja quais são os assuntos do momento no Yahoo! + Buscados: Top 
 10http://br.rd.yahoo.com/mail/taglines/mail/*http://br.maisbuscados.yahoo.com/-
 Celebridadeshttp://br.rd.yahoo.com/mail/taglines/mail/*http://br.maisbuscados.yahoo.com/celebridades/-
 Músicahttp://br.rd.yahoo.com/mail/taglines/mail/*http://br.maisbuscados.yahoo.com/m%C3%BAsica/-
 Esporteshttp://br.rd.yahoo.com/mail/taglines/mail/*http://br.maisbuscados.yahoo.com/esportes/



[obm-l] Problema de máximo!!!

2009-11-02 Por tôpico Pedro Júnior
Prove que, entre todos os triângulos retângulos de catetos a e b e
hipotenusa c fixada, o que tem maior soma dos catetos
S = a + b é o triângulo isósceles.


[obm-l] Re: [obm-l] resolução de provas....

2009-10-07 Por tôpico Pedro Júnior
Sempre sai nas EUREKA cara...
aguarda!!!

2009/10/7 ruy de oliveira souza ruymat...@ig.com.br

 A OBM não disponibilizou a resolução das provas da terceira fase, nível
 1,2,3 do ano de 2008. Onde acho as resoluções dos três niveis? alguém sabe
 de algum site?
Abraços



[obm-l] Re: [obm-l] [obm-l] RE: [obm-l] Teorema da Ordinalid ade dos Números Primos

2009-06-29 Por tôpico Pedro Júnior
Vamos lá Marco estou aguardando o material, afim de tentar compreender
algo...
Abraços e Parabéns

2009/6/29 Marco Bivar marco.bi...@gmail.com

 Caros Rhilbert/Felipe, obrigado pelas considerações. Olha, uma coisa eu
 digo a vocês: estou sendo sincero, não há motivos porque mentir(!). Se
 minhas técnicas parecem ser pouco convecionais, a culpa é minha, por
 esquecer de ler nos livros e confiar apenas no papel e lápis!

 Bem, o que afirmo é que a ordinalidade dos números primos é determinada
 daquela forma, i.e., fazendo a diferença p-c-1, pois p é tomado como número
 posicionado na sucessão dos números naturais (o que condiz) e c nada mais é
 do que o valor de posições deslocadas de compostos devido ao método de
 aproximar os números primos (ou seja, c é quantidade de compostos até p).

 Ainda, como temos Op=p-c-1, obtemos c=p-|Op|-1 e p=c+|Op|+1 (Pn=n+c+1).
 Isso é demasiadamente complexo computacionalmente. Podemos obter qualquer
 número primo, desde que saibamos calcular o c. Eu pergunto: nós sabemos? O
 que temos é o conjunto dos números p-complementares, do qual poderão surgir
 novas relações.

 Tudo bem, mas como eu calculei naqueles exemplos? Não disse que sabia como,
 afirmei que podemos fazê-lo. Como eu fiz? Simples: lancei mão de uma lista
 de números primos, ao lado de cada um sua ordem em valor absoluto. Como sei
 que c=p-|Op|-1, uma planilha funcionou bem. Mas como agora tenho a lista dos
 números p-complementares, então posso calcular qualquer número primo. Mas
 não sabemos como calcular c, realmente! Não sabendo determinar o c não
 podemos calcular o p!

 Enfim, para ser mais realista, digo o seguinte: se você conhece o primo p e
 a quantidade de números compostos c até p, e quer saber a ordem do primo p,
 faça |Op|=p-c-1. Se conhece um primo p e sua ordem, e quer saber quantos
 números compostos aparecem antes de p, faça c=p-|Op|-1 (esta, mais realista,
 pois conhecemos milhares de primos). Se conhece a quantidade de números
 compostos até um primo p e a ordem desse primo p e quer saber qual é o primo
 p, faça Pn=n+c+1. A essência está em sabermos como fazê-lo, não se iremos
 conseguir fazê-lo - conseguir fazer é coisa que deixo para os mais
 avançados. Talvez eu mesmo consiga resolver esse problema um dia e então
 ficarei feliz.

 Rhilbert, não coloco o c como a quantidade de compostos de zero até p-1
 (p-1 é número composto se não consideramos a exceção de 2 e 3), eu digo que
 c é o número que representa a quantidade de compostos até p. A redundância
 que você encontrou está em substituir |Op| pelo valor do outro membro em
 p=c+|p-c-1|+1. Não é assim que funciona. Isso é absolutamente redundante.
 Existem meios a serem descobertos para se chegar ao que você procura. Aliás,
 não estamos procurando meios para calcular os números primos? Igualmente.

 Agora, pessoalmente, acho que esta é a fórmula mais bonita para calcular
 números primos: é simples, elegante e funciona. Nem preciso dizer porque
 vocês são matemáticos e vocês também sabem mais ou menos porque eu sou!
 Uma coisa afirmo com a devida certeza: se um dia conseguirmos calcular os
 números primos e ainda tivermos a esperança de que isso aconteça numa
 simples equação, podemos esperar que isso vai ser em função do c na fórmula
 Pn=n+c+1, pois no resto da fórmula não será necessário mudanças.

 --
 Marco Bivar



Re: [obm-l] Conjuntos - Problema!!!

2009-04-05 Por tôpico Pedro Júnior
Um ótimo raciocínio
E, claro que ajudou!!!
Não é realmente bom o problema? Encontramos sempre problemas fáceis de
conjuntos, e esse não é tão bobinho..
Abraços colegas
2009/4/3 Hugo Fernando Marques Fernandes hfernande...@gmail.com

 Pedro.

 Seja P o número de participantes em cada conferência. Então 13P/12 pessoas
 assistiram somente a uma conferênciae (300 - 13P/12) assistiram a mais de
 uma. Sabendo que as três conferências foram assistidas pelo mesmo número de
 pessoas, a conferência com o maior número de pessoas dentre as (300 -
 13P/12) que assistiram mais de uma conferência será a terceira, que tem o
 menor número de pessoas que assistiram somente a ela. Assim, o número de
 pessoas na terceira conferência, P, será no máximo igual a P/4 + (300 -
 13P/12).

 Resolvendo a equação:

 P/4 + (300 - 13P/12) = P

 vem P = 163,636363...

 Então P  163,63 e pelo fato de 13P/12 ser um número inteiro positivo, P é
 múltiplo de 12.

 Ora, o maior múltiplo de 12 menor que 163,6363.. é 156.

 Espero ter ajudado.

 Abraços.

 Hugo.

 2009/4/2 Pedro Júnior pedromatematic...@gmail.com

 Ok, Alex, aora imagine você em um concurso onde terás em torno de 3,5 min
 por questões! Então, quando postei tal problema na lista esperei que fosse
 de interesse geral dicutir melhores saídas para resolver problemas, mas acho
 que a turma não leu tal questão, pois inicialmente parece um probleminha
 bobo de conjuntos, quando de fato não é!!!
 Se a banca elaboradoa do concurso colocou sabendo da média de tempo que o
 candidato tem por questão deve ter um caminho manífico por aí
 Vamos esperar e ver se alguém com mais experiância que nós dois
 colabora...
 Deixo aqui meus sinceros agradecimentos pela sua engenhosa colaboração
 Abraços!!!

 2009/3/26 Alex pereira Bezerra alexmatematica1...@gmail.com



 Seja P o número de pessoas que compareceram a cada uma das conferências. Do
 enunciado da questão e lembrando que o mesmo número de pessoas ( 
 *P*)compareceram às três conferências, poderemos escrever, com base na 
 figura
 acima:

 P/2 + x + y + t = P
 P/3 + x + y + z = P
 P/4 + x + z + t = P

 Lembrando que o número total de pessoas é igual a 300, é lícito escrever
 também:

 P/2 + P/3 + P/4 + x + y + z + t = 300

 Efetuando todas as operações indicadas em relação a P nas equações acima
 e arrumando, fica:

 x + y + t = P/2
 x + y + z = 2P/3
 x + z + t = 3P/4
 x + y + z + t = 300 – 13P/12

 Substituindo o valor de *x +  y + t* = P/2 na equação *x + y* + z *+ t*= 
 300 – 13P/12, fica:

 P/2 + z = 300 – 13P/12, de onde vem: z = 300 – 19P/12
 Substituindo o valor de *x + y + z* = 2P/3 na equação *x + y + z* *+ *t
 = 300 – 13P/12, fica:
 2P/3 + t = 300 – 13P/12, de onde vem: t = 300 – 21P/12

 Substituindo o valor de *x + z + t *= 3P/4 na equação *x* + y *+ z + t*= 
 300 – 13P/12, fica:
 3P/4 + y = 300 – 13P/12, de onde vem: y = 300 – 22P/12

 Substituindo os valores encontrados para y, z e t na equação
 x + y + z + t = 300 – 13P/12, vem:

 x + (300 – 22P/12) + (300 – 19P/12) + (300 – 21P/12) = 300 – 13P/12
 Desenvolvendo e simplificando a expressão acima, vem:
 x + 900 – 300 = 22P/12 + 19P/12 + 21P/12 – 13P/12
 x + 600 = 49P/12
 x = 49P/12 – 600

 Em resumo:
 x = 49P/12 – 600
 y = 300 – 22P/12
 z = 300 – 19P/12
 t = 300 – 21P/12

 Ora, como x, y, z *e* t  referem-se a quantidade de pessoas, serão
 necessariamente números inteiros e positivos ou seja: x  0, y  0,
 z  0 *e* t  0. Observe também que nas expressões de x, y, z e t acima,
 sempre aparece o valor P dividido por 12, ou seja,
 para que x, y, z e t sejam inteiros, P deverá ser necessariamente um
 múltiplo de 12.

 Então poderemos escrever:
 49P/12 – 600  0  , logo,  49P/12  600 , logo, 49P  7200 , logo, P 
 7200/49 e, portanto P  146,93

 Analogamente,
 300 – 22P/12  0 , logo, 300  22P/12  , logo,  22P/12  300  , logo, 22P
  3600 e, portanto P  163,63

 E, também,
 300 – 19P/12  0 , logo, 300  19P/12  , logo,  19P/12  300  , logo,   19P
  3600 e, portanto P  189,47

 E, finalmente,
 300 – 21P/12  0  , logo,  300  21P/12 , logo,  21P/12  300  , logo,
 21P  3600  , e , portanto P  171,42

 Logo, o valor de *P* tem que ser inteiro e múltiplo de 12 e atender
 simultaneamente às desigualdades P  146,93  e  P  163,63  e
 P  189,47  e P  171,42. Então, o valor de P é um número múltiplo de
 12, maior do que 146,93 e menor do que 163,63.
 A sucessão de inteiros que satisfazem à segunda condição é:
 147, 148, 149, 150, 151, 152, 153, 154, 155, *156*, 157, 158, 159, 160,
 161, 162 , 163.
 Destes, o único que é múltiplo de 12 é *156*, que é a resposta do
 problema, ou seja, a alternativa correta é a de letra C.






Re: [obm-l] Conjuntos - Problema!!!

2009-04-02 Por tôpico Pedro Júnior
Ok, Alex, aora imagine você em um concurso onde terás em torno de 3,5 min
por questões! Então, quando postei tal problema na lista esperei que fosse
de interesse geral dicutir melhores saídas para resolver problemas, mas acho
que a turma não leu tal questão, pois inicialmente parece um probleminha
bobo de conjuntos, quando de fato não é!!!
Se a banca elaboradoa do concurso colocou sabendo da média de tempo que o
candidato tem por questão deve ter um caminho manífico por aí
Vamos esperar e ver se alguém com mais experiância que nós dois colabora...
Deixo aqui meus sinceros agradecimentos pela sua engenhosa colaboração
Abraços!!!

2009/3/26 Alex pereira Bezerra alexmatematica1...@gmail.com



 Seja P o número de pessoas que compareceram a cada uma das conferências. Do
 enunciado da questão e lembrando que o mesmo número de pessoas ( 
 *P*)compareceram às três conferências, poderemos escrever, com base na figura
 acima:

 P/2 + x + y + t = P
 P/3 + x + y + z = P
 P/4 + x + z + t = P

 Lembrando que o número total de pessoas é igual a 300, é lícito escrever
 também:

 P/2 + P/3 + P/4 + x + y + z + t = 300

 Efetuando todas as operações indicadas em relação a P nas equações acima e
 arrumando, fica:

 x + y + t = P/2
 x + y + z = 2P/3
 x + z + t = 3P/4
 x + y + z + t = 300 – 13P/12

 Substituindo o valor de *x +  y + t* = P/2 na equação *x + y* + z *+ t* =
 300 – 13P/12, fica:

 P/2 + z = 300 – 13P/12, de onde vem: z = 300 – 19P/12
 Substituindo o valor de *x + y + z* = 2P/3 na equação *x + y + z* *+ *t =
 300 – 13P/12, fica:
 2P/3 + t = 300 – 13P/12, de onde vem: t = 300 – 21P/12

 Substituindo o valor de *x + z + t *= 3P/4 na equação *x* + y *+ z + t* =
 300 – 13P/12, fica:
 3P/4 + y = 300 – 13P/12, de onde vem: y = 300 – 22P/12

 Substituindo os valores encontrados para y, z e t na equação
 x + y + z + t = 300 – 13P/12, vem:

 x + (300 – 22P/12) + (300 – 19P/12) + (300 – 21P/12) = 300 – 13P/12
 Desenvolvendo e simplificando a expressão acima, vem:
 x + 900 – 300 = 22P/12 + 19P/12 + 21P/12 – 13P/12
 x + 600 = 49P/12
 x = 49P/12 – 600

 Em resumo:
 x = 49P/12 – 600
 y = 300 – 22P/12
 z = 300 – 19P/12
 t = 300 – 21P/12

 Ora, como x, y, z *e* t  referem-se a quantidade de pessoas, serão
 necessariamente números inteiros e positivos ou seja: x  0, y  0,
 z  0 *e* t  0. Observe também que nas expressões de x, y, z e t acima,
 sempre aparece o valor P dividido por 12, ou seja,
 para que x, y, z e t sejam inteiros, P deverá ser necessariamente um
 múltiplo de 12.

 Então poderemos escrever:
 49P/12 – 600  0  , logo,  49P/12  600 , logo, 49P  7200 , logo, P 
 7200/49 e, portanto P  146,93

 Analogamente,
 300 – 22P/12  0 , logo, 300  22P/12  , logo,  22P/12  300  , logo, 22P
  3600 e, portanto P  163,63

 E, também,
 300 – 19P/12  0 , logo, 300  19P/12  , logo,  19P/12  300  , logo,   19P
  3600 e, portanto P  189,47

 E, finalmente,
 300 – 21P/12  0  , logo,  300  21P/12 , logo,  21P/12  300  , logo, 21P
  3600  , e , portanto P  171,42

 Logo, o valor de *P* tem que ser inteiro e múltiplo de 12 e atender
 simultaneamente às desigualdades P  146,93  e  P  163,63  e
 P  189,47  e P  171,42. Então, o valor de P é um número múltiplo de 12,
 maior do que 146,93 e menor do que 163,63.
 A sucessão de inteiros que satisfazem à segunda condição é:
 147, 148, 149, 150, 151, 152, 153, 154, 155, *156*, 157, 158, 159, 160,
 161, 162 , 163.
 Destes, o único que é múltiplo de 12 é *156*, que é a resposta do
 problema, ou seja, a alternativa correta é a de letra C.




[obm-l] Conjuntos - Problema!!!

2009-03-26 Por tôpico Pedro Júnior
Esse probleminha foi, como estão vendo da prova de 2008 da universidade Fed.
de Campina rande  - PB
O lance é: como melhor discutir com nossos alunos?

Gabarito (c)

Abraços!!!

(UFCG - 2008) Em um ciclo de três conferências, que ocorreram em
horários distintos, havia sempre o mesmo número de pessoas assistindo
a cada uma delas. Ora, sabe-se que a metade dos que compareceram
à primeira conferência não foi a mais nenhuma outra; um terço dos
que compareceram à segunda conferência assistiu a apenas ela e um
quarto dos que compareceram à terceira conferência não assistiu nem
a primeira nem a segunda. Sabendo ainda que havia um total de 300
pessoas participando do ciclo de conferências, e que cada uma assistiu
a pelo menos uma conferência, o número máximo de pessoas em cada
conferência foi:
(a) 180.
(b) 80
(c) 156
(d) 210
(e) 96.


[obm-l] Material de Básica!!!

2009-02-27 Por tôpico Pedro Júnior
Boa noite a todos...
Queria saber se alguém têm ou conhece algum lugar em que possa baixar uma
boa lista de exercícios de Mat. Básica, principalmente problemas envolvendo
proporcionalidade, regra de três, porcentagem, etc...
Material com problemas simples tenho muito, queria na realidade problemas
mais complicados desse assunto, apesar de ser um assunto fácil por
natureza...
Agradeço antecipadamente.
Pedro Jr


Re: [obm-l] [OFF] perseguicao

2009-01-27 Por tôpico Pedro Júnior
Ei cara se desespere não!!!
Dá um pulinho aqui na Paraíba, que a galera da um jeitinho em vósmicê!!!
A turma aqui não rejeita nada, ninguém escapa, principalmente tatuado com o
número 7 de mentiroso, cabra de pêa!!!
Vai aprender a ser homem!!!

2009/1/23 João Luís joaolui...@uol.com.br

  Oi Leandro,

 Você poderia então esclarecer pra nós o que foi isso então...

 Pq, além de essa sua resposta ter sido a primeira séria (nada contra as
 outras, até gosto de piadas, principalmente as que tentam levar a coisa
 pruma situação menos beligerante) a respeito dessa) arespeito de  um troço
 totalmente descabido, já é a terceira vez que essa mensagem é postada aqui.

 Abraço,

 João Luís

 - Original Message -
 *From:* LEANDRO L RECOVA leandrorec...@msn.com
 *To:* obm-l@mat.puc-rio.br
 *Sent:* Friday, January 23, 2009 1:34 AM
 *Subject:* RE: [obm-l] [OFF] perseguicao

 Isso e uma ofensa ao Professor Terence Tao aqui de UCLA.

 Nao respondam essas mensagens.

 Regards,

 Leandro.


 --

 Date: Thu, 22 Jan 2009 22:21:06 -0300
 Subject: Re: [obm-l] [OFF] perseguicao
 From: fgam...@gmail.com
 To: obm-l@mat.puc-rio.br

 O primeiro colocado já está morto?


 2009/1/22 Felipe Diniz edward.elric.br@ edward.elric...@gmail.com
 gmail.com

 Respeitem o maior especialista do mundo em Formulas para Numeros Primos.


 On Thu, Jan 22, 2009 at 9:42 PM, João Maldonado 
 joao_maldona...@hotmail.com wrote:

 Cada uma...

  Date: Thu, 22 Jan 2009 06:24:17 -0800
  From: mathfire2...@yahoo.com.br
  Subject: [obm-l] [OFF] perseguicao
  To: obm-l@mat.puc-rio.br

 
  Ola,
 
  Venho comunicar que apos eu obter a setima colocacao brasileira
  na Olimpiada Iberoamericana de Matematica Universitaria em 2006
  fui perseguido das mais diversas formas. Fui preso e drogado de
  forma covarde, tendo serios danos a saude provacados por drogas
  que me obrigaram a tomar. Hoje tenho dificuldade para manter a
  atencao, para ler e para estudar. Venho tentando a meses denunciar
  esta situacao junto as autoridades, mas, ao que parece elas sao
  coniventes com o que esta ocorrendo.
 
  Peco a ajuda de algum membro da lista.
 
  A situacao esta insustentavel e suspeito que tentarao me matar,
  fazendo parecer um problema de saude meu ou um acidente, ou
  ainda fazendo parecer que o responsavel por minha morte seja eu
  mesmo.
 
  Abracos.
 
 
  Eric Campos
  =
  DEUS=MATEMATICA
  Eric Campos Bastos Guedes - O maior
  especialista do mundo em Formulas para
  Numeros Primos.
  Endereco: RUA DOMINGUES DE SA, 422
  ICARAI - NITEROI - RJ - CEP: 24220-091
  BRAZIL
  mathf...@gmail.com
  mathfire2...@yahoo.com.br
  MSN: fato...@hotmail.com
  =
 
 
 
  Veja quais são os assuntos do momento no Yahoo! +Buscados
  http://br.maisbuscados.yahoo.com
 
  =
  Instruções para entrar na lista, sair da lista e usar a lista em
  http://www.mat.puc-rio.br/~obmlistas/obm-l.htmlhttp://www.mat.puc-rio.br/%7Eobmlistas/obm-l.html
  =

 --
 É fácil compartilhar suas fotos com o Windows LiveT Arraste e 
 soltehttp://www.microsoft.com/windows/windowslive/photos.aspx






[obm-l] Problema - Olimpíada Matemática - PB

2008-10-19 Por tôpico Pedro Júnior
Problema da Olimpíada Pessoense de Matemática - 2008
Será que alguém poderia me ajudar na resolução desse problema:

1) Sabendo que cos(5x) = 16cos^5(x) - 20cos^3(x) + 5cos(x). Calcule cos18°.
Determine todos os valores reais positivos, x e y, que são soluções da
equação:
x^2 +4xcos(xy) + 4 = 0






pois estou sem perceber uma resolução que independa do uso de derivadas.


[obm-l] Funções

2008-09-29 Por tôpico Pedro Júnior
1) Encontre todas as funções tais que f(x2 + f(y)) = y + f(x)2.

Dica: prove que f(x2) = f(x)2 e que f(x + y) = f(x) + f(y) para x não
negativo e y real.

Olá pessoal...

Não estou conseguindo resolver esse problema, se posível me enviar uma
solução.

Desde já agradeço.

Pedro Jr


Re: [obm-l] Desigualdade Elementar

2008-08-15 Por tôpico Pedro Júnior
O propblema, da forma que propus pode parecer aberto a todas as ferramentas
de álgebra que conhecemos, mas da lista que tirei só podíamos resolver
usando algumas propriedades bem restritas, mas, mesmo assim acho que ficou
legal!
Essa questão se encontra no Cap. 0 do livro do Munem.
Abraços !
Quem tiver mais idéias favor postar por aqui, e muito obrigado!
Pedro Júnior

2008/8/14 Rafael Ando [EMAIL PROTECTED]

 é, acho que é melhor do que o que eu tinha proposto. legal :)


 On Thu, Aug 14, 2008 at 11:48 PM, Guilherme Leite Pimentel 
 [EMAIL PROTECTED] wrote:

 Acho que basta o seguinte (sx=sqrt[x])

 yx = y -x  0 = (sy-sx)(sy+sx)0. Como sy+sx é necessariamente
 positivo, segue que sy-sx0, de onde resulta a desiguldade.
 []s

 On Thu, Aug 14, 2008 at 5:09 AM, Rafael Ando [EMAIL PROTECTED]
 wrote:
  Bom, deve ter uma maneira mais elementar, mas acho que seria suficiente
  provar que a funcao sqrt(x) eh crescente, usando derivadas a
 derivada de
  sqrt(x) eh 1/2sqrt(x)  0, entao a funcao eh crescente
 
 
  On Wed, Aug 13, 2008 at 5:58 PM, Pedro Júnior 
 [EMAIL PROTECTED]
  wrote:
 
  Prove que se 0  x  y, ,então raiz(x)  raiz(y).
 
 
  --
  Rafael
 

 =
 Instruções para entrar na lista, sair da lista e usar a lista em
 http://www.mat.puc-rio.br/~obmlistas/obm-l.htmlhttp://www.mat.puc-rio.br/%7Eobmlistas/obm-l.html
 =




 --
 Rafael



[obm-l] Desigualdade Elementar

2008-08-13 Por tôpico Pedro Júnior
Prove que se 0  x  y, ,então raiz(x)  raiz(y).


[obm-l] Congruência!!!

2008-06-05 Por tôpico Pedro Júnior
01) Existe um inteiro positivo tal que seus fatores primos pertenem ao
conjunto {2, 3, 5, 7} e que terminam em 11? Se existir, ache o menor deles.
Se não existir, mostre porquê.

claramente percebe-se que tal problema poderá ser feito sem congruência,
mas, como esse problema faz parte de uma lista de exercícios de congruência
então, queria saber como faço...

Abraços a todos.


[obm-l] trigonometria

2008-05-29 Por tôpico Pedro Júnior
Boa noite a todos...
Me deparei com esse probleminha e ainda não consegui vê a saída!

Sabendo-se que sen1° .sen2°. sen3° . ... . sen85° .sen87° .sen89° = 1/2^n,
mostre que n45.

Acho que alguém mandou e minha esposa limpou miha caixa de e-mail's e a
solução foi junto, parece piada, mas foi o que aconteceu!
Se alguém tiver enviado e puder enviar novamente agradeço desde já!
Abraço a todos.


Re: [obm-l] equação

2008-04-27 Por tôpico Pedro Júnior
2001/11/1, Pedro [EMAIL PROTECTED]:

  Amigos da lista , me dê um idéia para essa equação:

 --
 [image: sin^{14}{x} + cos^{14}{x} = \frac {1}{64}]


Pedro fiquei curioso não com a questão, mas como vc fez para digitar essa
equação aqui, sei que usou o Latex, mas onde escreveu os comando?
Abraços
2181a1a59ab195dd3341a5c7802bbd4efacbba7e.gifattachment: 2181a1a59ab195dd3341a5c7802bbd4efacbba7e.gif

Re: [obm-l] Corolário

2008-04-13 Por tôpico Pedro Júnior
O que quis dizer quando..
Sendo 0  0 falso, segue-se que p(0) : 0  0 -- 0=1 é verdade.
Muito obrigado!!!

2008/4/13 Johann Peter Gustav Lejeune Dirichlet 
[EMAIL PROTECTED]:

 Na verdade, pelo que eu entendi, ele usa o princípio da boa ordem
 travestido de indução.

 Cê prova que de n+1 vale pra n, e prova que para n=1 falha. Deu pra
 entender?


 Em 06/04/08, Pedro Júnior[EMAIL PROTECTED] escreveu:
  Seguinte Bruno, o livro usa o axioma da indução para resolver, apenas
 não
  entendi como ele concluiu, como disse estou um pouco enferrujado, mas
 com um
  pouco de paciência vou conseguir
  ele usa,
  Seja S um subconjunto de N tal que
   i) 0 pertence a S
  ii) S é fechado com respeito à operação de somar 1 a seus elementos,
 ou
  seja,
  para todo n, se n pertence a S, então n+1 pertence a S
   Logo, S = N
  pois é como falei o livro usou isso, e não ficou tão claro algumas
 coisas,
  vejam:
  Corolário 1
  Não existe nenhum número natural n tal que 0  n  1.
  Demonstração: O enunciado acima é equivalente dizer que
 
  p(n) : n  0 --n=1
  é verdade para todo n pertencente aos Naturais.
 
  Sendo 0  0 falso, segue-se que p(0) : 0  0 -- 0=1 é verdade.
 
  Por outro lado, note que p(n+1) : n+1  0-- n+11 é verdade para todo n
  natural. De fato, n + 1 = 1 é verdade para todo n natural. Logo, sendo
  p(n+1) verdade para todo n, segue-se que p(n) -- p(n+1) é verdade para
 todo
  n natural.
   Portanto o resultado decorre do Princípio da Indução Matemática
 
  Prá quem está afiado o autor deve está sendo extremamente claro, só que
 prá
  mim não está tão claro...
 
  Então se alguém consegui entender claramente, pederia por favor que me
  esclarecesse, desde já agradeço bastante.
 
  Abraços
 
  Pedro Jr
 
  2008/4/4 Bruno França dos Reis [EMAIL PROTECTED]:
 
   De onde você quer partir? Quer dizer, quais axiomas vc quer admitir
 para
  demonstrar tal fato?
  
  
  
   2008/4/4 Pedro Júnior [EMAIL PROTECTED]:
  
  
  
  
Mostre que entre 0 e 1 não existe nenhum número natural.
   
Bom na realidade esse corolário está demonstrado no livro do Hefez,
  infelizmente não consegui entender tal demonstração, será que alguém
 poderia
  mmostrar de outra maneira ou me explicar o que claramente o autor quis
  dizer?
   
Agradeço antecipadamente.
   
Pedro Jr
   
  
  
  
   --
   Bruno FRANÇA DOS REIS
  
   msn: [EMAIL PROTECTED]
   skype: brunoreis666
   tel: +33 (0)6 28 43 42 16
  
   e^(pi*i)+1=0
 
 


 --
 Ideas are bulletproof.

 V

 =
 Instruções para entrar na lista, sair da lista e usar a lista em
 http://www.mat.puc-rio.br/~obmlistas/obm-l.htmlhttp://www.mat.puc-rio.br/%7Eobmlistas/obm-l.html
 =



Re: [obm-l] Soma !!!

2008-04-09 Por tôpico Pedro Júnior
Eita mundão da matemática...

Rapaz 1ª vez que vi esta fórmula, nossa, mas faz sentido claro...

vou verificar valeu mesmo, só uma perguntinha, onde vc encontrou essa
questão mesmo?

pois encontrei numa lista de exercício por aí, e coloquei na minha porém não
havia resolvido antes.

resultado nome da questão: UM PROFESSOR EM APUROS!!!

KKK

Bom, agradeço bastante a colaboração e vou apicar indução afim de verificar
se vale para todo n.

abraços

E a caminhada continua!

2001/11/1 Pedro [EMAIL PROTECTED]:

  Essa questão deu muito trabalho à tres semana, mais no fim deu certo.

   Seja S_n = 1.11^0 + 2.11^1 +3.11^2 +...+n.111 rescrever
 de uma maneira para facilitar a solução:

  S_n = 1.(10^1 - 1)/9 +2.(10^2 - 1)/9 ++n.(10^n - 1)/9

S_n = 1/9.[ *(1.10^1 +2.10^2+...+n.10^n)* - (1+2+3+...+n)]

   Esta parte que eatá em negrito é : Série aritmético - geométrica. Você
 aplica a sguinte fórmula:

   S_n=[ a_1(1 - q^n)/1- q]   + rq[1 - nq^(n - 1) +(n - 1).q^n]/(1
 - q)^2

 obs:a_0=0 , a_1=1 e q=10

   Portanto,

   *S_n= 1/9 {10/81( 1+9n.10^n - 10^n) - [n(n+1)]/2}*

   Testei com n=1,2,3 e deu certo


 - Original Message -
 *From:* saulo nilson [EMAIL PROTECTED]
 *To:* obm-l@mat.puc-rio.br
 *Sent:* Tuesday, April 08, 2008 11:26 PM
 *Subject:* Re: [obm-l] Soma !!!

 (1+n)n/2+(2+n)(n-1)/2+(3+n)(n-3)/2,,,
 soma(k+n)(n-(k-1))/2=1/2soma(n^2-k^2)+n+k=
 =1/2(n^3+n^2+(1+n)n/2-n(n+1)(2n+1)/6=
 =3n(n+1)(6n+3-(2n+1))=12n(n+1)^2

 2008/4/8 Pedro Júnior [EMAIL PROTECTED]:

  Engalhei na seguinte soma:
 
  Já usei aquele exercício do livro do Lidisk, mas aquela soma é de 1 + 11
  + 111 + ... + (111...1), onde (111...1) tem exatamente n dígitos, mas mesmo
  assim ainda não saiu!
 
 
  S_n  =  1 + 22 + 333 +  + ... + n ( 111...1)
 
 
  onde (111...1) tem exatamente n dígitos.
 
  Desde Já agradeço!!!
 




[obm-l] Soma !!!

2008-04-08 Por tôpico Pedro Júnior
Engalhei na seguinte soma:

Já usei aquele exercício do livro do Lidisk, mas aquela soma é de 1 + 11 +
111 + ... + (111...1), onde (111...1) tem exatamente n dígitos, mas mesmo
assim ainda não saiu!


S_n  =  1 + 22 + 333 +  + ... + n ( 111...1)


onde (111...1) tem exatamente n dígitos.

Desde Já agradeço!!!


Re: [obm-l] Lista séries

2008-04-08 Por tôpico Pedro Júnior
dá uma olhadinha no site de um professor da UFPB ele tem um livro publicado
por uma editora internacional, um dos melhores professores que já vi, acho
que no site dele tem algumas coisa...

o nome do professor é Marivaldo P. Matos
e sua página é:

 www.mat.ufpb.br/matos


2008/4/8 Luiz Guilherme [EMAIL PROTECTED]:

 Ola pessoal,

 Alguém tem alguma lista ou conhece algum site que tenha uma lista
 analisando a convergência de algumas séries ja conhecidas?

 Abraços

 --
 Abra sua conta no Yahoo! 
 Mailhttp://br.rd.yahoo.com/mail/taglines/mail/*http://br.mail.yahoo.com/,
 o único sem limite de espaço para armazenamento!



Re: [obm-l] Corolário

2008-04-06 Por tôpico Pedro Júnior
Seguinte Bruno, o livro usa o axioma da indução para resolver, apenas não
entendi como ele concluiu, como disse estou um pouco enferrujado, mas com um
pouco de paciência vou conseguir
ele usa,
Seja S um subconjunto de N tal que
i) 0 pertence a S
ii) S é fechado com respeito à operação de somar 1 a seus elementos, ou
seja,
para todo n, se n pertence a S, então n+1 pertence a S
Logo, S = N
pois é como falei o livro usou isso, e não ficou tão claro algumas coisas,
vejam:
Corolário 1
Não existe nenhum número natural n tal que 0  n  1.
Demonstração: O enunciado acima é equivalente dizer que
p(n) : n  0 --n=1
é verdade para todo n pertencente aos Naturais.

Sendo 0  0 falso, segue-se que p(0) : 0  0 -- 0=1 é verdade.

Por outro lado, note que p(n+1) : n+1  0-- n+11 é verdade para todo n
natural. De fato, n + 1 = 1 é verdade para todo n natural. Logo, sendo
p(n+1) verdade para todo n, segue-se que p(n) -- p(n+1) é verdade para todo
n natural.
Portanto o resultado decorre do Princípio da Indução Matemática

Prá quem está afiado o autor deve está sendo extremamente claro, só que prá
mim não está tão claro...

Então se alguém consegui entender claramente, pederia por favor que me
esclarecesse, desde já agradeço bastante.

Abraços

Pedro Jr

2008/4/4 Bruno França dos Reis [EMAIL PROTECTED]:

 De onde você quer partir? Quer dizer, quais axiomas vc quer admitir para
 demonstrar tal fato?


 2008/4/4 Pedro Júnior [EMAIL PROTECTED]:

 Mostre que entre 0 e 1 não existe nenhum número natural.
 
  Bom na realidade esse corolário está demonstrado no livro do Hefez,
  infelizmente não consegui entender tal demonstração, será que alguém poderia
  mmostrar de outra maneira ou me explicar o que claramente o autor quis
  dizer?
 
  Agradeço antecipadamente.
 
  Pedro Jr
 



 --
 Bruno FRANÇA DOS REIS

 msn: [EMAIL PROTECTED]
 skype: brunoreis666
 tel: +33 (0)6 28 43 42 16

 e^(pi*i)+1=0


Re: [obm-l] Inteiros!!!

2008-04-06 Por tôpico Pedro Júnior
Falou nobre amigo, que Deus continue lhe dando sabedoria...
Abraços


2008/4/4 Paulo Santa Rita [EMAIL PROTECTED]:

 Ola Pedro e demais
 colegas desta lista ... OBM-L,

 Obviamente que todo par (X,Y) da forma (X,-X) e solucao, pois :
 X^3 + (-X)^3 = 0 = (X+(-X))/2.  Em particular, (0,0) e solucao.

 Se, porem, X+Y # 0, teremos :
 X^3 + Y^3 = (X+Y)*(X^2 -XY + Y^2) = (X+Y)/2. = X^2 - XY + Y^2 = 1/2
 = (X-Y)^2 = - (X^2 +Y^2) .
 A possibilidade aqui, logicamente, e :  X-Y=0 e X^2+Y^2 = 0. Mas isso
 da (X,Y)=(0,0)
 o que contraria a hipotese X+Y # 0

 Assim, todas as solucoes inteiras sao {(X,-X) / X e inteiro }

 Um Abracao a Todos
 Paulo Santa Rita
 6,0A2D,040408

 2008/4/4 Pedro Júnior [EMAIL PROTECTED]:
  02. Ache todos os pares tais de números inteiros (x, y) tais que:
  x^3 + y^3 = (x + y)^2

 =
 Instruções para entrar na lista, sair da lista e usar a lista em
 http://www.mat.puc-rio.br/~obmlistas/obm-l.htmlhttp://www.mat.puc-rio.br/%7Eobmlistas/obm-l.html
 =



[obm-l] Corolário

2008-04-04 Por tôpico Pedro Júnior
Mostre que entre 0 e 1 não existe nenhum número natural.

Bom na realidade esse corolário está demonstrado no livro do Hefez,
infelizmente não consegui entender tal demonstração, será que alguém poderia
mmostrar de outra maneira ou me explicar o que claramente o autor quis
dizer?

Agradeço antecipadamente.

Pedro Jr


[obm-l] Inteiros!!!

2008-04-03 Por tôpico Pedro Júnior
02. Ache todos os pares tais de números inteiros (x, y) tais que:
x^3 + y^3 = (x + y)^2


Re: [obm-l] Casas em Praça

2008-04-03 Por tôpico Pedro Júnior
Extremamente criativa a idéia das tiras, muito boa mesmo!
Agradeço!

Em 03/04/08, [EMAIL PROTECTED] [EMAIL PROTECTED] escreveu:

 Inadvertidamente, apaguei o e-mail cujo problema era:

 Existem casas em volta de uma praça. Rodrigo e Juan dão uma volta na
 praça, caminhando no mesmo sentido e contando as casas. Como não começaram
 a contar da mesma casa, a 5ª casa de Rodrigo é a 12ª casa de Juan, e a 5ª
 casa de Juan é a 30ª de Rodrigo. Quantas casas existem em volta da praça?

 Imaginem-se duas tiras circulares de iguais tamanhos com os números
 naturais nelas inscritos.
 A circularidade pode ser substituída pela repetição da retificação. Eis
 a figura:

 Rodrigo:  301 2 3  4  5
 Juan: 5 6 7 8 9 10 11 12

 Naturalmente, são 32 casas.


 =
 Instruções para entrar na lista, sair da lista e usar a lista em
 http://www.mat.puc-rio.br/~obmlistas/obm-l.html
 =



Re: [obm-l] teoria dos inteiros-ajuda

2008-04-02 Por tôpico Pedro Júnior
olá colega acho que sua expressão matemática não está escrita corretamente!
Pois sendo a = 0, temos apenas b...
ou não entendi...
reescreva por favor, ou me ajude a entender..
Abraços

2008/3/27, [EMAIL PROTECTED] [EMAIL PROTECTED]:

 a é inteiro, b inteiro não nulo, tais que (2^n).a + b é um quadrado
 perfeito
 para todo n natural. Prove que a=0.



 =
 Instruções para entrar na lista, sair da lista e usar a lista em
 http://www.mat.puc-rio.br/~obmlistas/obm-l.html
 =



  1   2   >